Anda di halaman 1dari 49

PAKET 1 C.

Penggunaan smart phone memberikan


dampak positif dalam poses belajar-
Teks berikut digunakan untuk menjawab soal mengajar.
nomor 2 dan 3. D. Pengaruh Teknologi smart phone
memberikan dampak positif dan negatif
(1) Perkembangan teknologi yang semakin
bagi generasi saat ini.
canggih khususnya smart phone, memberikan
E. Penggunaan smart phone dikalangan
pengaruh bagi generasi saat ini. (2) Hal ini
remaja.
dilihat dari penggunaan smart phone dikalangan
remaja. (3) Penggunaan smart phone, dapat
2. Kalimat utama padaparagraf ke-2 terletak
mendukung proses belajar-mengajar sehingga
pada kalimat ke ...
proses transfer of knowledge dan pembinaan
A. (9)
karater dan keterampilan belajar lancar. (4)
B. (10)
Namun, kita juga sering menjumpai remaja yang
C. (11)
berada dalam sebuah forum tanpa
D. (12)
berkomunikasi satu dengan yang lain, karena
E. (13)
asyik dengan dunianya sendiri. (6) Meminjam
bahasa Don Tapscott (2013), generasi ini Teks berikut digunakan untuk menjawab soal
adalah generasi acuh tak-acuh. (7) Minat nomor 3 dan 4.
mereka hanya mengenai budaya populer, para
pesohor, dan teman-teman mereka. (8) Pada (1) Semua orang pasti mengenal pendidikan.
dasarnya, teknologi digital membawa sejumlah (2) Pendidikan adalah proses internalisasi
dampak positif dan negatif. budaya ke dalam diri seseorang dan masyarakat
sehingga membuat orang dan masyarakat jadi
(9) Menurut Felder dan Soloman (1993), beradab. (3) Pendidikan bukan hanya
“Pembelajar di zaman informasi ini mempunyai merupakan sarana transfer ilmu penngetahuan,
kecenderungan gaya belajar aktif, sequential, tetapi lebih luas lagi, yakni sebagai sarana
sensing, dan visual.” (10) Fokus pembelajara pembudayaan dan penyaluran nilai
adalah pembelajaran seumur hidup, bukan demi (enkulturisasi dan sosialisasi). (4) Anak harus
ujian semata. (11) Guru tidak perlu khawatir jika mendapatkan pendidikan yang menyentuh
siswa lupa tanggal peristiwa penting dalam dimensi dasar kemanusiaan. (5) Dimensi
sejarah, karena mereka dapat mencarinya kemanusiaan itu mencakup sekurang-kurangnya
melalui buku dan web. (12) Guru perlu tiga hal paling mendasar.
mengajari mereka cara belajar yang baik dan
mendorong mereka untuk gemar membaca dan (6) Pendidikan karakter adalah pendidikan
menulis. (13) Jadi, yang terpenting bukan hanya budi pekerti plus, yaitu yang melibatkan aspek
tentang apa yanng diketahui ketika mereka pengetahuan, perasaan, dan tindakan. (7)
lulus, melainkan juga untuk mencintai Menurut Lickona, tanpa ketiga aspek itu,
pembelajaran seumur hidup. pendidikan karakter tidak akan efektif. (8)
Dengan pendidikan karakter yang diterapkan
1. Apa ide pokok paragraf ke-1 pada teks secara sistematis dan berkelanjutan, seorang
diatas? anak akan menjadi cerdas cerdas emosinya. (9)
A. Pengaruh teknologi bagi generasi saat Kecerdasan emosi ini adalah bekal penting
ini. dalam mempersiapkan anak menyongsong
B. Smart phone memberikan pengaruh masa depan. (10) Terdapat sembilan pilar
bagi remaja. karakter yang berasal dari nilai-nilai luhur
universal, yaitu karakter cinta Tuhan dan
segenap ciptaan-Nya; kemandirian dan Indonesia dewasa ini. Fenomena masyarakat
tanggung jawab; kejujuran/amanah dan semacam ini tampaknya sudah dipahami dan
diplomatis; hormat dan santun; dermawan, suka disadari pemerintah. Mentri Pendidikan dan
menolong dan gotong royong/kerja sama; Kebudayaan menegaskan adanya kerisauan
percaya diri dan pekerja keras; kepemimpinan dan kerinduan banyak pihak untuk kembali
dan keadilan; baik rendah hati; serta toleran dan memperkuat pendidikan karakter dan budaya
cinta damai. (SBMPTN 2013) bangsa tersebut melalui pendidikan sekolah.

3. Manakah gagasan utama paragraf ke-2 ? 5. Masalah utama paragraf di atas adalah ....
A. Pendidikan karakter yang efektif. (SBMPTN 2014)
B. Pendidikan merupakan sarana A. Kemerosotan karakter masyarakat
pembudayaan. Indonesia.
C. Pendidikan karakter merupakan B. Penanaman karakter melalui
pendidikan yang melibatkan pendidikan di sekolah.
pengetahuan, perasaan, dan tindakan. C. Program pemerintah dalam pendidikan
D. Pendidikan harus berdimensi karakter.
kemanusiaan. D. Sosialisasi pendidikan karakter di
E. Pendidikan dikenal setiap orang. sekolah.
E. Pentingnya penguatan karakter dan
budaya bangsa.
4. Apa perbedaan gagasan antarparagraf
dalam teks tersebut? Pulau Penyengat yang berpendudukan
A. Paragraf pertama memaparkan 1.000 jiwa itu memiliki nama lain, yaitu
pendidikan karakter secara umum, Maskawin, karena pulau tersebut merupakan
sedangkan paragraf kedua mas kawin Sultan Mahamudsyah, raja
memaparkan pendidikan karakter kedelapan Kesultanan Riau Lingga ketika ia
secara khusus. mempersunting Enngku Putri Raja Hamidah.
B. Paragraf pertama memaparkan konsep
pendidikan karakter, sedangkan 6. Gagasan utama kalimat tersebut adalah ...
paragraf kedua memaparkan unsur A. Pulau Peyengat berpenduduk 1.000
pendidikan karakter. jiwa..
C. Paragraf pertama memaparkan B. Pulau Penyengat merupakan mas
pendidikan secara umum, sedangkan kawin Sultan Mahmudsyah.
paragraf kedua memaparkan tujuan C. Sultan Mahmudsyah adalah raja
pendidikan secara khusus. kedelapan Kesultanan Riau Lingga.
D. Paragraf pertama memaparkan konsep D. Pulau Penyengat memiliki nama lain.
pendidikan secara umum, sedangkan E. Sultan Mahmudsyah mempersunting
paragraf kedua memaparkan konsep Engkau Putri Raja Hamidah.
pendidikan karakter.
E. Paragraf pertama memaparkan 7. Ide pokok dalam kalimat majemuk tersebut
pendidikan secara umum, sedangkan adalah .... (SIMAK UI 2015)
paragraf kedua memaparkan manfaat A. Pelaku bisnis mulai banyak.
pendidikan karakter. B. Pelaku bisnis memasarkan produk
dan layanan.
Karakteristik dan perilaku tidak terpuji yang C. Pelaku bisnis yang memasarkan
menerpa masyarakat sudah merupakan bagian produk dan layanan.
yang tidak terpisahkan dari perilaku masyarakat
D. Pelaku bisnis memasarkan produk A. Kemunduran kesenian Betawi
dan layanan secara online. B. Seni pertunjukan budaya Betawi
E. Pelaku bisnins tertarik pada C. Pelestarian budaya Betawi
kepraktisan, jangkauan pasar, D. Kondisi budaya Betawi
kemudahan akses, dan cara E. Seni musik tradisional Betawi.
pembayaran.
9. Gagasan utama paragraf selanjutnya pada
Teks berikut digunakan untuk menjawab soal teks tersebut adalah ....
nomor 8 dan 9. A. Kebudayaan Betawi yang tidak
dinikmati masyarakat.
(1) Saat ini makin banyak kebudayaan
B. Kebudayaan Betawi yang megalami
Betawi yang tidak dapat dinikmati masyarakat
kemunduran.
globalisasi. (2) Contohnya adalah seni
C. Upaya meningkatkan kelstarian
pertunjukkan ondel-ondel. (3) seni pertunjukkan
kebudayaan Betawi.
ondel-ondel adalah seni pertunjukkan denga
D. Faktor-faktor yang mempengaruhi
boekabesar setinggi dua meter. (4) pada
kondisi kebudayaan Betawi
awalnya, boneka ini berfungsi sebagai penolak
E. Dampak globalisasi terhadap
bala dari gangguan roh halus. (5) dalam
kebudayaan Betawi.
perkembangan selanjutnya, pertujukan ondel-
ondel juga digunakan untuk menambah
semarak pesta-pesta rakyat, peresmian gedung
baru, atau menyambut tamu terhormat. Akhir-akhir ini, banyak pemuda berperilaku
kebarat-baratan dengan memakai anting-anting
(6) Contoh lain kebudayaan Betawi yang di hidungnya. Perilaku tersebut merupakan
terancam tergerus globalisasi adalah Lenong. fenomena yang menarik utuk diperhatikan dan
(7) Lenong merupakan teater tradisional Betawi dibahas. Sebagian orang menilai bahwa
yang diiringi musik gambang kromong. (8) fenomena tersebut sebagai simbol kepedulian
Gambang kromong adalah musik tradisional global yang dilakukan oleh para pemuda
Betawi yang mendapat pengaruh dari suku peduduk dunia saat ini. Di negara Barat, bukan
Tionghoa. (9) Lakon atau scenario lennong hanya pemuda, melainkan juga beberapa orang
umumya mengandung pesan moral, seperti dewasa memakai anting-anting di hidung,
membela kaum lemah. (10) Pertunjukan itu telinga, pipi, atau dibagian tertentu dari anggota
biasanya diadakan di lapangan terbuka tanpa badannya. Karena negara Barat diidentikkan
panggung. (11) ketika pertunjukan tersebut dengan budaya maju, para pemuda di negara
berlangsung, salah seorang aktor atau aktris lain, terutama negara berkembang, ikut-ikutan
mengitari penonon untuk meminta sumbangan melakukannya supaya dikatakan sebagai
secara sukarela. (12) pada acara-acara di pemuda modern yang mengikuti mode saat ini.
panggung hajatan, seperti resepsi pernikahan.
(13) Selain kedua seni pertunjukan tersebut, 10. Ide pokok paragraf diatas adalah ...
budaya arsitektur dan pertanian Betawi juga A. Pemakaian anting-anting.
mengalami kemunduran. (14) Conntohnya B. Fenomena mode para pemuda.
rumah-rumah asli Betawi di kawasan Condet C. Simbol kepedulian global.
saat ini terabaikan. (15) Tambahan lagi, rumah- D. pemuda di zaman modern.
rumah baru orang betawi saat ini tidak E. Sikap pemuda yang kebarat-baratan.
menggunakan gaya arsitektur Betawi.
11. Kalimat topik pada paragraf di atas
8. Gagasan pokok bacaan di atas adalah... adalah ....
A. Pengaruh budaya Korea di Indonesia. 13. Ide pokok paragraf tersebut adalah ....
B. Masuknya budaya Korea di Indonesia. A. Budaya gotong royong
C. Indonesia merupakan negara Korean B. Peralatan penanan padi
Wave. C. Penekanan biaya produksi
D. Layar televisi Indonesia menayangkan D. Perekat rasa persaudaraan
drama kora, film korea, dan musik E. Tradisi masyarakat Dayak Jalai
korea.
E. Indonesia termasuk negara yang (1) Sepanjang tahun 2011, bank-bank
sedang terkena demam korea. nasional mampu menikmati kenaikan laba cukup
besar. (2) Namun, bank-bank asing yang
Pada dasarnya tujuan pendidikan adalah beroperasi di Indonesia malah bernasib beda.
membentuk kepribadian peserta didik. (3) Sejumlah bank harus rela memperoleh laba
Pembetukan tersebut dapat dilakukan dengan pada tahun 2011 lebih kecil dibandingkan tahun
model langsung dan inkulkasi. Model langsung 2010.
adalah pembentukan moral melalui bidang ajar
yang terkait langsung dengan pendidikan (4) Mengutip laporan keuangan publikasi
karakter, seperti agama dan budi pekerti, bank di situs Bank Indonesia (BI), tiga
sedangkan model inkulkasi adalah penanaman perusahaan mencatat penurunan laba. (5) Laba
nilai-nilai pendidikan karakter melalui bidang- Cathy Bank menurun sehingga melambatnya
bidang yang tidak terkait langsung, seperti bisnis perusahaan pasca sanksi BI di divisi
bidang sains, teknologi, sosial, seni, dan olah konsumen. (6) Pendapatanpun turun, sementara
raga. Idealnya, setiap mata pelajaran beban usaha meningkat.
memengaruhi cara berpikir dan bertindak
peserta didik sehingga ikut membentuk karakter. 14. Ide pokok paragraf selajutnya pada teks di
atas adalah ....
12. Masalah utama paragraf di atas adalah .... A. Upaya penanganan turunnya laba.
A. Pembentukan kepribadian peserta didik B. Penyebab menurunnya laba.
B. Penanaman karakter melalui C. Akibat turunnya laba.
pendidikan di sekolah D. Masalah turunnya laba
C. Model-model pendidikan karakter di E. Perusahaan yang mengalami
sekolah penurunan laba.
D. Perlunya pendidikan karater disekolah-
sekolah 15. Kalimat pejelas yang tepat untuk
E. Pengaruh karakter terhadap cara melanjutkan kalimat di atas adalah ...
bepikir peserta didik. A. Hal ini dikarenakan, pendidikan
merupakan persyaratan yang paling
(1) Gotong royong saat berladang masih
utama untuk melamar pekerjaan.
membudaya di komunitas Dayak Jalai,
B. Melalui pendidikan yang berbasis
Kalimantan Barat. (2) Mereka bekerja sukarela
keahlian, seperti vokasi, dihararapkan
tanpa diganjar upah sepeser pun. (3) Pemilik
dapat menjadi alternatif untuk
ladang cukup mensediakan bahan dan
mengatasi jumlah pegangguran di
peralatan penanam padi serta konsumsi. (4)
Indonesia.
Bekerja dengan gotong royong saat
C. Namun, ada juga yang menilai dari
berladangtidak semata mampu menekan biaya
tingkat keahliannya.
produksi. (5) Tradisi itu juga merekatkan rasa
persaudaraan dan solidaritas warga.
D. Semakin tinggi tingkat pendidikan maka
semakin besar peluang untuk
mendapatkan pekerjaan.
E. Banyaknya pelamar yang tidak memiliki
ijazah mengakibatkan mereka sulit
untuk mendapatkan pekerjaan.
PAKET 2 merupakan yang tercanggih
(E) produk SS tidak hanya canggih,
Teks berikut digunakan untuk menjawab soal 1-2 tetapi juga terjangkau harganya
Teks 3A
(1) Perhelatan Forum SS kembali 2. Informasi manakah yang terdapat pada
digelar di Kuala Lumpur Convention Centre, Teks 3A, tetapi tidak terdapat pada Teks
Malaysia. (2) Acara ini bertujuan 3B?
memperkenalkan produk terbaru SS yang
(A) forum SS digelar di Kuala Lumpur
dipasarkan di Asia Tenggara. (3) SS telah
Convention Center
menjadi brand dengan pertumbuhan yang
(B) kulkas canggih dengan sistem
cepat di kategori aplikasi rumah tangga dan
operasi Android
ponsel pintar. (4) SS memantau keinginan
(C) mesin cuci yang dikontrol ponsel
konsumen untuk menghadirkan produk-
yang canggih
produk yang memudahkan kehidupan
(D) kecanggihan produk rumah tangga
manusia. (5) Produk SS yang hadir di tahun
SS
ini semuanya terkoneksi. (6) Produk-produk
(E) kecanggihan produk ponsel pintar SS
tersebut membangun ekosistem yang mampu
menghubungkan produk melalui jaringan
internet. 3. Bacalah teks berikut!
Teks 3B (1) Flu singapura pada dasarnya
Peserta forum diajak ke ruang pameran adalah flu yang menyerang kekebalan tubuh,
tempat seluruh amunisi SS menampilkan terutama anak-anak, yang dilakukan oleh virus
produk unggulan. (2) Salah satu produk yang RNA dari jenis Reterovirus. (2) Virus ini
mendapat sorotan adalah kulkas canggih. (3) pertama kali ditemukan di Singapura dan itu
Kulkas yang diotaki oleh sistem operasi pula yang kemudian menyebabkan flu ini
Android ini mampu mengakomodasi dinamakan flu singapura. (3) Dalam bahasa
kebutuhan keluarga dengan cara modern. (4) medis internasional, flu ini dinamakan HMFD
Selain itu, ada pula televisi terbaru yang atau Hand Mouth and Foot Disease. (4)
dilengkapi dengan teknologi Quantum Dot. (5) Penamaan ini berdasar pusat serangan yang
Teknologi ini diklaim menawarkan kualitas terfokus pada luka di kaki, tangan, dan mulut.
gambar realistis dengan desain yang semakin (5) Bila kebanyakan virus flu
menawan. (6) Perangkat lainnya adalah menyerang area pernafasan dan area lendir,
mesin cuci generasi terbaru yang dapat maka serangan virus RNA cenderung lebih
dikontrol lewat perangkat ponsel. muncul pada kulit. (6) Kadang gejala pada kulit
akan muncul pertama kali dari gejala demam
dan tanda-tanda flu lain, seperti munculnya
1. Apa simpulan kedua teks tersebut? sariawan berat dan peradangan di area
(A) SS merupakan perusahaan tenggorokan depan. (7) Proses penularannya
teknologi terdepan di Asia cukup mudah, tetapi kebanyakan penularan
(B) semua produk SS yang berawal dari interaksi langsung kulit dengan
dipamerkan terhubung satu sama lain media yang sudah terpapar virus RNA. (8)
(C) produk SS canggih dan Penularan itu bisa melalui proses sentuhan
memudahkan kehidupan manusia kulit dengan pengidap flu singapura atau
(D) kulkas produk SS tersentuh benda yang sudah terkontaminasi flu
singapura. (9) Aktivitas makan dan minum kota-kota besar. (8) Permainan
bercampur dengan pengidap flu singapura juga tradisional pada umumnya memerlukan
bisa menyebabkan penularan. arena luas, umpamanya bermain
(Diadaptasi dari http://deherba.com/apa-obat- kelereng, bermain gasing, petak umpet,
herbal-flu-singapura-terbaik.html/) dan lain-lain. (9) Selain itu, banyak
orang tua melarang anaknya bermain
permainan tradisional karena takut kotor
atau takut kulit anaknya terbakar panas
Pernyataan manakah yang sesuai dengan isi
matahari. (10) Orangtua banyak
teks di atas?
memberikan kepada anaknya
(A) Orang dewasa cenderung tidak permainan elektronik, misalnya video
terserang flu singapura. game dan mobil-mobilan. (11)
(B) Penularan virus flu singapura melalui Permainan-permainan tersebut
kontak langsung. dimainkan di dalam rumah saja. (12)
(C) Singapura menjadi tempat Akibatnya, anak kurang bersosialisasi
penyebaran flu. dengan temannya dan kurang bergerak.
(D) Flu singapura menyerang kulit dan
tenggorokan.
4. Pernyataan mana yang TIDAK
(E) Flu singapura bisa ditularkan melalui
sesuai dengan isi teks?
udara.
(A) Permainan tradisional sangat baik
Jawaban: B
untuk kesehatan anak.
Catatan: Perhatikan kata kunci pada paragraf (B) Permainan tradisional memiliki
kedua kata interaksi langsung sinonim dengan banyak manfaat
kata kontak langsung (C) Permainan tradisional tidak
mudah dimainkan di kota-kota
besar.
(D) Permainan tradisional sangat
Bacalah teks berikut untuk menjawab soal no 7-
bermanfaat untuk kegiatan sosial
8!
anak.
(1) Permainan tradisional dilakukan (E) Permainan tradisional banyak
dengan banyak gerakan oleh anak-anak, didukung para orangtua.
misalnya permainan kasti, gasing, dan
kelereng. (2) Dengan demikian, dia akan
5. Apa pesan tersirat di balik teks yang
terhindar dari obesitas, (3) Sosialisasi dan
ingin disampaikan penulis?
komunikasi mereka tercapai, karena dalam
(A) Permainan tradisional
permainan tradisional paling sedikit
mengindarkan anak dari penyakit
dimainkan oleh dua anak. (4) Permainan
obesitas
tradisional juga dapat menentukan strategi
(B) Permainan tradisonal melatih
dalam bermain. (5) Mereka juga akan
kemampuan komunikasi dan
bekerja sama dengan anggota tim. (6)
sosialisasi anak
Permainan tradisional bagi anak sangat baik.
(C) Permainan tradisional melatih
kerjasama anak
(7) Permainan tradisional saat ini (D) Permainan tradisional tidak mudah
tidak mudah dilakukan, terutama di dilakukan
(E) Permainan tradisional perlu perkembangan daerah ini dimulai pada
dilestarikan karena memiliki banyak usia yang sangat muda. (11) Yang lebih
manfaat menarik adalah bagaimana aktivitas
otak di wilayah ini lebih sibuk pada
anak-anak yang orang tuanya gemar
6. Bacalah teks berikut! membaca. (12) Membacakan buku
(1) Sebuah studi menunjukkan untuk anak membantu pertumbuhan
bahwa anak yang dibiasakan neuron di daerah ini yang akan
mendengarkan cerita sejak dini dan menguntungkan anak di masa depan
dikenalkan dengan kebiasaan membaca dalam hal kebiasaan membaca.
memiliki perkembangan jaringan otak
yang lebih awal. (2) Sebaliknya, anak
yang tidak dikenalkan dengan
kebiasaan membaca memiliki (Diadaptasi dari
perkembangan yang kurang pada http:health.kompas.com/
jaringan tersebut. (3) Anak-anak balita read/2016/02/07/135500623/ memba-
dengan orang tua yang rutin cakan.Buku.Meningkatkan.Kinerja.Ot
membacakan buku untuk mereka ak.Balita)
mengalami perbedaan perilaku dan
prestasi akademik dengan anak-anak
dengan orang tua yang cenderung pasif
dalam membacakan buku. (4) Menurut
sebuah studi baru yang diterbitkan Apa simpulan teks tersebut?
dalam jurnal Pediatrics menemukan
(A) Anak-anak yang belajar membaca
perbedaan yang juga terjadi pada
pada usia 3-5 tahun akan
aktivitas otak anak.
mempercepat perkembangan
(5) Peneliti mengamati perubahan
otaknya.
aktivitas otak anak-anak usia 3 sampai
(B) Semakin awal kebiasaan membaca
dengan 5 tahun yang mendengarkan
buku diperkenalkan, semakin aktif
orang tua mereka membacakan buku
otak anak bekerja.
melalui scanner otak yang disebut
(C) Anak-anak yang mulai belajar
functional magnetic resonance imaging
membaca sejak balita akan menjadi
(FMRI). (6) Orang tua menjawab
anak-anak yang otak kirinya lebih
pertanyaan tentang berapa banyak
aktif.
mereka membacakan cerita untuk anak-
(D) Kebiasaan mendengarkan cerita
anak serta seberapa sering melakukan
dan membaca sejak usia balita
komunikasi. (7) Para peneliti melihat
berdampak positif untuk
bahwa ketika anak-anak sedang
perkembangan otak.
mendengarkan orang tua bercerita,
(E) Otak kiri bertanggung jawab
sejumlah daerah di bagian kiri otak
terhadap proses pemahaman
menjadi lebih aktif. (8) Ini adalah daerah
seperti kata dan konsep.
yang terlibat dalam memahami arti kata,
konsep, dan memori. (9) Wilayah otak
ini juga menjadi aktif ketika anak-anak Bacalah teks berikut untuk menjawab soal
bercerita atau membaca. (10) Pada no 7-8!
studi ini menunjukkan bahwa
(1) Globalisasi menjadi tantangan untuk 8. Apa pesan tersurat dari teks tersebut?
semua aspek kehidupan termasuk (A) Tips menghadapi globalisasi
kebudayaan. (2) Era global menuntut (B) Dampak globalisasi terhadap
kesiapan kita untuk siap berubah industri makanan
menyesuaikan perubahan zaman dan mampu (C) Globalisasi menuntut kesiapan
mengambil setiap kesempatan. (3) Budaya manusia untuk siap berubah
tradisional di Indonesia sebenarnya lebih (D) Pentingnya menyaring informasi di
kreatif dan tidak bersifat meniru, namun yang era global
menjadi masalah adalah bagaimana (E) Cermat menghadapi globalisasi
mempertahankan jati diri bangsa. (4) Sebagai
contoh sederhana, budaya gotong royong di
Indonesia saat ini hampir terkikis habis, yang 9. Bacalah teks berikut!
digantikan oleh sikap individual dan tidak (1) Penggunaan istilah Empat Pilar
peduli kepada orang lain. (5) Perlu dipikirkan pada hakikatnya ingin meneguhkan
agar kebudayaan kita tetap dapat Pancasila sebagai dasar berbangsa dan
mencerminkan kepribadian bangsa. bernegara. (2) Pancasila tetaplah dasar
(1) Dalam era globalisasi, kebudayaan negara meski pada saat bersamaan
tradisional mulai mengalami erosi. (7) Semua dilekatkan dengan istilah semisal Pancasila
orang, terutama anak muda lebih senang sebagai pandangan hidup berbangsa,
menghabiskan waktunya mengakses internet Pancasila adalah perjanjian luhur bangsa,
daripada mempelajari tarian dari kebudayaan dan lain sebagainya. (3) Bung Karnopun
sendiri. (8) Orang akan merasa bangga ketika sebagai penggali Pancasila pernah
dapat meniru gaya berpakaian orang Barat memaknainya sebagai bintang penuntun
dan menganggap budayanya ketinggalan arah (leadstar) perjuangan bangsa. (4) Ini
zaman. (9) Globalisasi akan selalu menunjukkan, betapa banyak keragaman
memberikan perubahan, sehingga harus fungsi atau peran Pancasila bagi bangsa
diteliti apakah berbagai budaya yang masuk dan negara kita. (5) Keragaman makna
tersebut bersifat positif atau negatif. seperti itu tentu tak akan menggoyahkan
(Diadaptasi dari http://www.budayatradisional kedudukan Pancasila sebagai dasar
indonesia.blogspot.co.id/) Negara. (6) Wacana Pancasila sebagai
dasar atau bukan itu sudah lama usai. (7)
Kita jangan lagi kembali ke belakang.

7. Kalimat manakah yang merupakan


simpulan dan isi teks di atas? Rangkuman yang tepat untuk wacana di atas
(A) Budaya tradisional Indonesia adalah….
orisinal dan kreatif.
(B) Kebudayaan tradisional perlu (A) Istilah Empat Pilar pada hakikatnya ingin
dipertahankan. meneguhkan Pancasila.
(C) Globalisasi menggerus kebudayaan (B) Pancasila tetaplah dasar meski
tradisional. diletakkan dengan istilah yang beragam.
(D) Budaya gotong royong di Indonesia (C) Penggunaan istilah Empat Pilar sebagai
terkikis habis. dasar atau bukan sudah lama usai.
(E) Generasi muda mengidolakan (D) Penggunaan istilah Empat Pilar tidak
budaya Barat akan menggoyahkan negara
berpancasila.
(E) Penggunaan istilah Empat Pilar tidak Desa Mindi, Kecamatan Porong,
akan menggoyahkan kedudukan Kabupaten Sidoarjo. Semburan itu
Pancasila sebagai dasar negara. merupakan semburan ke-59 yang muncul
di sekitar pusat semburan utama.
Menurut seorang ahli dan Leader Team
10. Bacalah teks berikut! Fergaco, perusahaan yang mengawasi
Selama bertahun-tahun terjadi banyak gas-gas berbahaya di sekitar lokasi
masalah kesehatan karena kekurangan air semburan, semburan itu sama dengan
bersih. Pendidikan minim karena kondisi 58 semburan liar sebelumnya. Semburan
ekonomi memprihatinkan. Sekarang, air bersih liar itu juga tidak berbahaya dan tidak
tidak sulit lagi. Setiap rumah sudah mempunyai akan membesar. Kalau dibiarkan
kamar mandi dan WC. Kehidupan ekonomi semburan itu akan mengecil sendiri.
warga terus membaik. Semua itu berakhir Untuk menutup semburan, hari ini akan
dengan pulihnya tiga mata air yang mengering dimasukkan 100 kilogram semen ke
setelah perusakan hutan tahun 1964 - 1965. dalam lubang asal semburan.
Dengan demikian, reboisasi Hutan Winosadi
membawa perubahan yang tak terbayangkan Simpulan yang tepat untuk paragraf tersebut
bagi masyarakat sekitar hutan tersebut. adalah…..

(A) Semburan lumpur, air, dan gas baru


yang keluar di Desa Mindi tidak
Simpulan isi paragraf tersebut adalah …. berbahaya
(B) Semburan yang keluar di Desa Mindi
(A) Perusakan hutan membawa dampak
merupakan semburan liar yang ke-59
negatif di segala sektor kehidupan
(C) Warga Desa Mindi menemukan
bagi masyarakat sekitar hutan
semburan lumpur liar di belakang
tersebut.
rumah seorang warga
(B) Kekurangan air bersih merupakan
(D) Semburan liar yang ke-59 tersebut
masalah besar bagi kesehatan
berbahaya
masyarakat sekitar hutan Wonosadi
akibat perusakan hutan. (E) Untuk menutup semburan liar yang
baru diperlukan 100 kilogram semen
(C) Reboisasi Hutan Wonosadi membawa
perubahan positif di bidang kesehatan
12. Bacalah teks berikut!
dan ekonomi bagi masyarakat sekitar.
(1)Kenaikan harga minyak dunia
(D) Ketersediaan air bersih adalah hal
telah membuat selisih harga bahan bakar
penting bagi peningkatan kehidupan
minyak (BBM) subsidi dan nonsubsidi
ekonomi setiap warga sekitar Hutan
semakin melebar. (2) Keadaan ini, yang
Wonosadi.
memicu peningkatan peralihan konsumsi
(E) Pulihnya mata air yang mengering
dari BBM nonsubsidi ke BBM bersubsidi.
selama bertahun-tahun membawa
(3) Tidak mengherankan jika realisasi
dampak positif bagi perekonomian
subsidi BBM hingga akhir tahun
masyarakat.
inidiperkirakan menembus angka 41 juta
kiloliter dari rencana yang dipatok APBN-P
11. Bacalah teks berikut!
2011 yang hanya 40,4 juta kiloliter.
Semburan lumpur, air, dan gas
(4) Kondisi tersebut tentu tidak
baru keluar dari halaman belakang
menguntungkan karena APBN akan
rumah salah seorang penduduk, warga
terbebani. (5) Subsidi BBM juga
menciptakan ketidakadilan karena (A) Kenaikan harga minyak dunia
penikmat subsidi BBM adalah kelompok membebani APBN karena kebijakan
masyarakat menengah ke atas. (6) Di sisi subsidi BBM lebih banyak efek
lain, alokasi subsidi BBM yang besar negatifnya sehingga pembatasan
menyebabkan anggaran belanja untuk penggunaan BBM bersubsidi
sector lainnya (termasuk bagi kelompok semestinya segera direalisasikan.
warga miskin) menjadi berkurang. (7) (B) Kebijakan pengurangan subsidi BBM
Apalagi, dengan tidak adanya dalam APBN merupakan langkah
pengendalian konsumsi BBM bersubsidi yang bijak tidak hanya demi keadilan,
juga menimbulkan efek negatif lainnya tetapi juga untuk menjaga kesehatan
yang tidak kalah berat. (8) BBM bersubsidi energy terbarukan di masa
menimbulkan pemborosan dalam hal mendatang
konsumsi energi takterbarukan. (9) kondisi (C) Kebijakan pembatasan BBM
ini sama saja dengan mempercepat bersubsidi merupakan langkah bijak
proses terjadinya kelangkaan energi. demi keadilan guna mewujudkan
(10) BBM bersubsidi hanya kebijakan membagi bebas subsidi
menguntungkan pemilik kendaraan pribadi BBM dari kelas menengah atas ke
sehingga menimbulkan persoalan pelik kelas bawah
bagi sistem transportasi darat di kota-kota (D) Kenaikan harga minyak dunia
besar. (11) Akibatnya, industry membebani APBN sehingga
transportasi missal menjadi tidak menarik, diperlukan langkah bijak demi
yang sebenanrnya diharapkan dapat keadilan karena yang menikmati
memecahkan masalah keruwtan subsidi BBM justru ada di kota-kota
transportasi darata. (12) Di sisi lain, BBM besar dan golongan kaya.
bersubsidi menimbulkan ketidakadilan (E) APBN akan jelas-jelas terbebani
antarwilayah, karena penikmat subsidi karena konsumsi BBM bersubsidi
BBM justru adanya di kota-kota besar dan pada tahun mendatang semakin
golongan kaya. besar sehingga berpotensi
(13) Jelaslah, mempertahankan menimbulkan implikasi negatif yang
kebijakan subsidi BBM sebenarnya lebih lebih berat.
banyak efek negatifnya dibandingkan
dengan manfaatmya. (14) Oleh karena
itulah, kebijakan pembatasan penggunaan
BBM bersubsidi semestinya segera 13. Bacalah teks berikut!
direalisasikan. (15) Apabila tidak dibatasi, Batuk sebetulnya bukan penyakit
dikhwatirkan konsumsi BBM yang refleks, karena mengalami rangsangan udara
tentunya berpotensi menimbulkan yang berpolusi, asap pabrik, asap rokok, bau-
implikasi negative yang lebih berat. (16) bauan, gas yang merangsang atau kekurangan
Kini di tangan pemerintah bola untuk udara. Batuk juga dapat terjadi karena saluran
menetapkan opsi pengaturan penggunaan pernapasan atau paru-paru terkena infeksi
BBM bersubsidi berada. kuman-kuman tertentu. Udara dingin atau
lembab dapat juga membuat orang batuk atau
Rangkuman manakah yang paling tepat bersin. Obat batuk yang dijual di pasar bebas
untuk teks tersebut? umumnya terdiri atas obat atau campuran obat
yang mengandung bahan yang dapat
mengeluarkan lendir atau riak agar saluran
pernapasan bersih dari gangguan atau Bersaudara Medan, yakni bola basket, senit tari,
rangsangan penyebab batuk itu, Obat batuk melukis, menyanyi, dan seni peran
jenis ini disebut ekspektoran.

Ringkasan teks yang tepat adalah ...


Ringkasan teks yang tepat adalah ...
(A) SMA Bina Bersaudara Medan
(A) Penyakit batuk dikarenakan membagikan formulir identitas untuk
rangsangan udara yang tidak berpolusi, mengetahui kegemaran tiap siswa.
asap rokok, gas yang merangsang atau Kegemaran utama siswa SMA Bina
kekurangan udara terinfeksi, maka Bersaudara Medan yakni bola basket,
barus diobati dengan ekspektoran. seni tari, melukis, menyanyi, dan seni
(B) Pengaruh batuk itu terjadi karena asap peran.
pabrik, gas asap rokok, udara yang (B) Diketahui ada lima jenis kegemaran
berpolusi, sehingga saluran paru-paru utama siswa SMA Bina Bersaudara
terinfeksi. Untuk mengobati penyakit Medan, yakni bola basket, senit tari,
tersebut dapat dibeli di pasar bebas. melukis, menyanyi, dan seni peran.
(C) Batuk terjadi karena polusi udara, asap (C) Dari formulir yang dibagikan itu dapat
pabrik, rokok, bau-bauan, gas, dikenali berbagai bakat dan minat
kekurangan udara saluran pernapasan. siswa. Hal itu dilakukan agar sekolah
Batuk dapat diobati dengan dapat membina bakat dan minat
ekspektoran. siswa secara benar.
(D) Terjadinya penyakit batuk karena (D) Meskipun tidak semua bakat dan
polusi udara, asap pabrik, bau-bauan, minat siswa itu terpenuhi di sekolah,
kekurangan udara. Namun, semua pihak sekolah patut mengetahui
dapat diobati dengan obat-obat kegemaran siswa. Sehingga
altematif yang dijual di pasaran bebas. dibagikanlah formulir kepada tiap
(E) Batuk adalah penyakit refleks dan siswa.
obatnya dapat diperoleh di pasar bebas (E) Melalui survey itu, pihak SMA Bina
Bersaudara Medan dapat mengetahui
kegemaran tiap siswanya.
14. Bacalah teks berikut!
Untuk menyalurkan bakat dan minat
siswa SMA Bina Bersaudara Medan, kepada 15. Bacalah pernyataan berikut!
siswa dibagikan formulir identitas guna
mengetahui kegemaran tiap siswa. Dari formulir Buaya dan kadal binatang melata.
yang dibagikan itu dapat dikenali berbagai bakat Ular dan buaya suka makan katak.
dan minat siswa. Hal itu dilakukan agar sekolah
dapat membina bakat dan minat siswa secara Simpulan yang tepat adalah….
benar. Meskipun tidak semua bakat dan minat (A) Binatang melata suka makan katak
siswa itu terpenuhi di sekolah, pihak sekolah (B) Kadal tidak suka makan katak
patut mengetahui kegemaran siswa. Dengan (C) Kadal suka makan katak
cara itu, sekolah dapat mencarikan wadah untuk (D) Kadal belum tentu makan katak
menyalurkan bakat dan minat siswa. (E) Ular merupakan binatang melata
Berdasarkan isian tersebut, diketahui ada lima
jenis kegemaran utama siswa SMA Bina
PAKET 3 menulis buku yang terjual paling banyak
tentang pengalaman terbangnya dan
1. (1) Rosan P.Roeslani, Ketua Kadin sebagai penolong formasi Ninety-Nine,
Indonesia, meyakini adanya peningkatan organisasi pilot wanita. (5) Earhart hilang
ekonomi Indonesia 2017. (2) Secara lebih secara misterius di samudera Pasifik di
spesifik, dia menilai investasi di sektor dekat Pulau Howland dalam usaha untuk
manufaktur bisa berperan banyak-setelah melakukan penerbangan keliling dunia
konsumsi dalam negeri dan belanja tahun 1937.
pemerintah-untuk mendorong
perekonomian demostik. (3) Dari sisi Inti kalimat (1) pada bacaan di atas
investasi, pandangan Rosan Jelas sejalan adalah_____
dengan keinginan besar Presiden Joko
A. Amelia Earhart adalah penerbang,
Widodo untuk mendorong investasi pada
pengarang, dan pilot.
tahun mendatang.(4) Pada 2017,
B. Amelia Earhart adalah penerbang
pemerintah menargetkan investasi
perempuan pertama Amerika Serikat.
mencapai Rp 670 triliun dan melonjak lagi
C. Amelia Earhart adalah penerbang yang
pada 2018 menjadi Rp 840 triliun. (5) Hal itu
hilang secara misterius.
tidaklah berlebihan bila melihat tren
D. Amelia Earhart adalah penerbang
investasi saat ini.
perempuan pertama yang hilang.
E. Amelia Earhart adalah hilang ketika
Inti kalimat (2) ____
penerbangan keliling dunia.
A. Investasi di sektor manufaktur bisa
berperan
3. Gedung bertingkat yang akan segera
B. Investasi berperan mendorong
direnovasi itu masih digunakan kuliah
perekonomian domestik
bersama mahasiswa antarjurusan di
C. Perekonomian domestik lebih
fakultas ini.
menopang investasi
Inti kalimat di atas adalah____
D. Dia menilai investasi di sektor
A. Gedung itu akan direnovasi
manufaktur
B. Gedung itu digunakan
E. Sektor manufaktur bisa berperan
C. Gedung itu digunakan kuliah bersama
banyak
D. Gedung itu digunakan kuliah bersama
antarjurusan
E. Gedung itu akan digunakan kuliah
2. (1) Amelia Earhart adalah seorang
bersama
penerbang, pengarang dan pilot perempuan
pertama Amerika Serikat yang hilang
4. Kini lembaga kependudukan dan KB yang
secara misterius di atas samudera Pasifik
pada masa Orde Baru merupakan badan
ketika melakukan penerbangan keliling
bergengsi yang didukung anggaran dan
dunia pada tahun 1937. (2) Earhart adalah
fasilitas prima itu terpinggirkan dan tidak
wanita pertama yang menerima
dianggap sebagai tempat bergengsi, karena
Distinguished Flying Cross.
tidak menjanjikan jenjang karier yang jelas
(3) Ia mendapat penghargaan itu,
bagi birokrat. Inti kalimat tersebut adalah
karena ia adalah wanita pertama yang
____
terbang sendiri menyeberangi samudera
Atlantik. (4) Ia juga membuat rekor lainnya
A. lembaga kependudukan dan KB pada diakses dari mana saja dan kapan saja,
masa Orde Baru merupakan badan serta cara pembayaran yang lebih mudah.
bergengsi Inti kalimat dari kalimat majemuk di atas
B. lembaga kependudukan dan KB yang adalah___
pada masa Orde Baru merupakan
A. pelaku bisnis mulai banyak
badan bergengsi didukung anggaran
B. pelaku bisnis memasarkan produk
dan fasilitas prima
dan layanan
C. lembaga kependudukan dan KB tidak
C. pelaku bisnis yang memasarkan
menjanjikan jenjang karier yang jelas
produk dan layanan
D. lembaga kependudukan dan KB
D. pelaku bisnis memasarkan secara
terpinggirkan dan tidak dianggap
online
sebagai tempat bergengsi
E. pelaku bisnis tertarik pada
E. lembaga kependudukan dan KB
kepraktisan, jangkauan pasar,
terpinggirkan dan tidak dianggap
kemudahan akses dan cara
sebagai tempat bergengsi, karena tidak
pembayaran
menjanjikan karier yang jelas

5. Sesuai Kesepakatan, PUPUK KALTIM akan


8. Udara yang kotor karena debu ataupun asap
kembali menyuplai gas pada Mei 2010
sisa pembakaran menyebabkan kadar
Inti Kalimat tersebut adalah____
oksigen berkurang sehingga sangat
A. Kesepakatan Pupuk Kaltim
membahayakan kelangsungan hidup setiap
B. Pupuk Kaltim menyuplai gas
organisme.
C. Suplai gas dilakukan oleh Pupuk Kaltim
Kalimat di atas merupakan perluasan dari
D. Kesepakatan Pupuk Kaltim untuk
kalimat dasar___
menyuplai gas pada Mei 2010
A. Udara kotor berkurang.
E. Suplai gas pupuk kaltim akan dilakukan
B. Udara kotor menyebabkan oksigen
pada Mei 2010
berkurang.
C. Asap menyebabkan oksigen.
6. Akibat vertical speed yang terlalu tinggi, D. Udara kotor membahayakan.
pesawat menjadi sulit dikontrol saat roda E. Asap sisa pembakaran
hendak menyentuh landasan sehingga membahayakan.
benturan keras terjadi dan pesawat pun
bisa terpental ke luar landasan. 9. Pemerintah sebagai penanggung jawab
Kalimat inti dari kalimat di atas adalah____ kesejahteraan rakyat memiliki tanggung
A. vertical speed terlalu tinggi jawab besar dalam upaya memikirkan dan
B. pesawat sulit dikontrol mewujudkan kelestarian lingkungan hidup.
C. roda pesawat menyentuh landasan
D. terjadi benturan keras Kalimat di atas merupakan perluasan dari
E. pesawat terpental ke luar landasan kalimat dasar ____
A. pemerintah penanggung jawab
7. Seiring kemajuan teknologi, mulai banyak kesejahteraan
pelaku bisnis yang memasarkan produk B. pemerintah memiliki tanggung jawab
dan layanan secara online karena C. tanggung jawab pemerintah
kepraktisan yang ditawarkan, jangkauan mewujudkan kelestarian
pasar tanpa batas, kemudahan untuk
D. pemerintah memikirkan kelestarian E. merehabilitasi korban merupakan salah
lingkungan satu cara memutus mata rantai pasar
E. pemerintah mewujudkan kesejahteraan obat-obatan terkutuk itu

12. Gabungkanlah kalimat-kalimat berikut


10. Kalimat yang polanya tidak sama dengan menjadi sebuah kalimat!
kalimat yang lain adalah .... 1. Pariwisata merupakan penghasil devisa
A. banyak buku dan tulisan mampu yang sangat diharapkan.
mengubah nasib sebuah bangsa 2. Wisatawan mencanegara merosot tajam
B. kegiatan membaca akan dapat tahun ini.
membangun karakter masyarakat 3. Teror merebak di mana-mana di seluruh
C. setiap pelajar diharuskan tanah air.
menghasilkan sebuah tulisan pendek
D. membaca tidak cukup dijadikan Pengabungan yang tepat adalah___
sebagai ajakan atau imbauan A. Pariwisata merupakan penghasil
E. pasti akan menemukan buku yang devisa yang sangat diharapkan, tetapi
tepat wisatawan mancanegara merosot
tajam tahun ini karena teror merebak di
11. Pasal 128 Undang-Undang Nomor 35 mana-mana di seluruh tanah air.
Tahun 2009 tentang Narkotika B. Pariwisata merupakan penghasil
menyebutkan, pecandu narkotik yang telah devisa yang sangat diharapan, dan
cukup umur dan tengah menjalani wisatawan mancanegara merosot
rehabilitasi medis di rumah sakit dan/atau tajam tahun ini karena teror merebak di
lembaga rehabilitasi medis yang ditunjuk mana-mana di seluruh tanah air.
pemerintah tidak dituntut pidana. C. Pariwisata merupakan penghasil
Kalimat yang memiliki pola kalimat yang devisa yang sangat diharapkan, tetapi
sama dengan kalimat di atas adalah ____ wisatawan mancanegara merosot
A. Dari sekian banyak kabar tajam tahun ini sehingga teror merebak
penggerebekan kelas teri itu, untunglah di mana-mana di seluruh tanah air.
terbetik kabar pembongkaran sindikat D. Pariwisata merupakan penghasil
sabu internasional oleh BNN devisa yang sangat di harapkan, tetapi
B. lembaga ini menangkap Wong Ching wisatawan mancanegara merosot
Ping, penyelundup 800 kilogram sabu- tajam tahun ini apalagi teror merebak di
sabu lewat Kepulauan Seribu, DKI mana-mana di seluruh tanah air.
Jakarta E. Pariwisata merupakan penghasil divisa
C. memenjarakan para pengguna narkoba yang sangat diharapkan, padahal
bukanlah pilihan terbaik, karena di wisatawan mancanegara merosot
penjara justru kian terbuka pergaulan tajam tahun ini karena teror merebak di
para korban itu dengan produsen dan mana-mana di seluruh tanah air.
pengedar yang ada di sana
D. Menteri Koordinator Perekonomian
mengatakan, pemerintah akan 13. Kalimat yang menyatakan kalimat majemuk
menaikkan angka penyertaan modal setara adalah____
negara (PMN) ke BUMN A. Dia tidak ragu sedikitpun dalam
menentukan pasangan hidupnya
B. Febi berbelanja sayuran dan Yayank
memasaknya
C. Anak itu rajin belajar bahkan dia
menjadi juara umum di sekolahnya
D. Terbukti teori darwin justru
mengacaukan seluruh ilmu dasar
dalam IPTEK
E. Apabila perekonomian Indonesia
kembali bangkit, secara otomatis
kesejahteraan rakyatnya pun akan
meningkat

14. Pola Kalimat Dian memilih sepatu dan Dani


membayarnya. Sama dengan pola kalimat
majemuk berikut ini, kecuali____
A. Ayah membaca koran, atau ayah
membaca majalah
B. Ibu tinggal disini, atau ibu ikut pulang
C. Leon dan Davi membentuk sebuah
band alternatif
D. Semua murid aktif mengikuti diskusi,
hanya Deni yang pasif.
E. Kakak menyapu halaman dan adik
membersihkan kamar mandi

15. Nelayan Menangkap ikan di laut


Kalimat berikut memiliki pola kalimat yang
sama dengan kalimat di atas, kecuali___
A. Mereka menonton konser musik
dilapangan
B. Kemarin Polisi menangkap pencuri
dipasar itu
C. Dia bergembira pada pesta ulang
tahunnya
D. Kau harus mengambalikan buku itu
pada tempatnya semula
E. Murid Membaca buku di perpustakaan
PAKET 4 B. Kami turut prihatin maka terjadinya
musibah di daerah itu
1. Bacalah kalimat-kalimat berikut!
a. Nilai luhur budaya kian terkikis C. Kami turut prihatin dari terjadinya
musibah di daerah itu
b. Seekor beruang kutub dilestarikan
di gurun
D. Kami turut prihatin supaya
c. Lia dan Mila sering bertengkar
terjadinya musibah di daerah itu
d. Barista meramu racikan kopi di
café E. Kami turut prihatin atas terjadinya
e. Dinda berjalan menggunakan musibah di daerah itu
sepatu baru
Kalimat yang tidak efektif ditandai 5. Pada pilihan kalimat di bawah ini, kata
dengan huruf… yang penulisannya tidak baku adalah…
A. Pemimpin berperilaku baik menjadi
A. a dan c panutan bawahannya
B. a dan e
B. Konflik antarkelompok menjadi
C. b dan c
D. b dan d sesuatu yang lazim
E. c dan d C. Sesama manusia harus
bekerjasama
2. Pembangunan infrastruktur tidak hanya D. Dia membasuh wajahnya dengan
membatasi pada wilayah pusat air
perkotaan. Kalimat tersebut tidak efektif E. Gerakan tubuh penari itu lemah
karena penggunaan kata yang salah.
gemulai
Kata yang salah ialah…
A. Infrastruktur 6. (1) Pada setiap adanya pembaruan
B. Hanya kurikulum, pihak dinas pendidikan
C. Perkotaan dituntut secara cepat melakukan
D. Wilayah penyesuaian terhadap para pelaksana
E. Membatasi di lapangan. (2) Akan tetapi, kenyataan
berbicara lain karena umumnya
3. Di bawah ini yang merupakan kalimat
penanganan pengembangan tenaga
efektif adalah…
A. Para hadirin dipersilakan pelaksana di lapangan berjalan sangat
memasuki ruangan lambat.
B. Hadirin dimohon untuk duduk lagi Ejaan pada kalimat (1) dan (2) di atas
kembali jikan diperbaiki menjadi ejaan yang
C. Hadirin hadirat dimohon berdiri benar dengan cara…
D. Hadirin dimohon berdiri A. Menulis kata ‘dinas pendidikan’
E. Saudara-saudara diharap tenang
dengan huruf awal capital
B. Menghilangkan tanda koma
4. Kami turut prihatin kepada terjadinya
setelah kata ‘akan tetapi’
musibah di daerah itu.
C. Menghilangkan tanda koma
Kalimat efektif hasil perbaikan kalimat setelah kata ‘kurikulum’
di atas adalah.... D. Mengubah kata ‘secara’ menjadi
cara
A. Kami turut prihatin sehingga E. Menambahkan tanda koma
terjadinya musibah di daerah itu setelah kata ‘lain’
7. Bacalah paragraf di bawah ini! Indonesia menjadi peringatan bahwa
Gubernur DKI Jakarta mengubah manusia dan hewan dari luar negeri
strategi dalam membangun perlu pengawasan. (4) Busung lapar
LRT. Tahap awal mempercayakan hanya puncak es dari kondisi gizi buruk
pihak swasta mengelola dua koridor. yang terabaikan. (5) Sikap
Setelah itu, Pemprov DKI Jakarta akan menyalahkan rakyat atau pemerintah
melanjutkan pembangunan koridor dalam mengatasai flu burung tidaklah
berikutnya dari hasil keuntungan dua bijaksana.
koridor awal. Jika untung, nanti lima
koridor akan dibiayai sendiri. Kalimat yang sumbang pada paragraf
Kalimat bercetak miring dalam paragraf tersebut adalah nomor...
tersebut tidak tepat karena…
A. Ketidak lengkapan unsur kalimat A. 1
berupa subjek
B. Ketidak tepatan penempatan unsur B. 2
kalimat predikat
C. Ketidak tepatan penempatan unsur C. 3
kalimat subjek
D. penggunaan unsur kalimat secara D. 4
berlebihan
E. ketidak lengkapan unsur kalimat E. 5
berupa keterangan
10. Bacalah paragraf berikut dengan teliti!
8. Para kordinator tiap divisi
merencanakan program kerja untuk Aku baru saja turun di terminal kota
satu periode kepengurusan. Makassar dengan sebuah tas besar.
Ketidakbakuan kalimat tersebut terletak Aku berasal dari kampung dan ingin
pada penggunaan kata… mencari pekerjaan di kota ini. Begitu
A. Kordinator yang seharusnya ditulis keluar dari gerbang terminal aku
koordinator lanngsung berada di pintu utama mall
B. Divisi yang seharusnya ditulis yang megah. Pemandangan seperti ini
defisi tidak ada di kampungku. Akhirnya
C. Program yang seharusnya ditulis satpam menghadangku dan
perogram mengamankanku. Aku dikira tersangka
D. Periode yang seharusnya ditulis pembawa senjata tajam, padahal itu
priode hanya pisau dapur.
E. Kepengurusan yang seharusnya
ditulis pengurus Kalimat yang tepat untuk memperbaiki
kalimat sumbang yang tercetak miring
9. Cermati teks berikut! adalah…

(1) Era globalisasi tidak terbendung A. Aku pertama kali ke mall untuk
lagi. (2) Mobilitas penduduk, barang, berbrlanja
dan hewan yang cukup tinggi memiliki
dampak negatif terhadap kesehatan. B. Satpam mall memeriksa dan
(3) Kasus flu burung yang menghantui menemukan pisau di tasku
C. Di mall ini pengamanan sangat Kalimat yang tepat untuk memperbaiki
ketat kalimat penutup surat lamaran
pekerjaan tersebut adalah…
D. Setiap selesai berbelanja di mall A. Besar harapan saya, Bapak
harus melapor satpam mempertimbangkan lamaran saya.
Atas kebijaksanaannya, diucapkan
E. Satpam memperhatikanku sedari terima kasih.
B. Besar harapan saya, Bapak dapat
tadi
menerima saya. Atas
kebijaksanaan Bapak, saya
11. Bacalah teks berikut! mengucapkan terima kasih.
C. Mohon Bapak dapat menerima
(1)Cuaca akhir-akhir ini memang saya. Atas kebijaksanaannya
sangat ekstrem dirasakan masyarakat. kepada saya, saya ucapkan terima
(2)Udara di siang hari yang sangat kasih.
D. Sudilah kiranya Bapak menerima
panas membuat orang menjadi gerah.
saya. Atas kebijaksanaan Bapak,
(3) Berdasarkan informasi dari BMKG dihaturkan terima kasih.
cuaca ekstrem ini disebabkan oleh E. Atas kebijaksanaan menerima
memanasnya suku muka laut di saya di perusahaan Bapak, saya
perairan Indonesia. (4) Selain itu, mengucapkan terima kasih.
pengaruh La Nina yang masih
berlangsung hingga saat ini juga 13. Atas perhatiannya kami ucapkan
menjadi fator cuaca ekstrem. banyak-banyak terima kasih
(5)Kondisi ini membuat masyarakat Perbaikan yang tepat untuk penutupan
menyerbu tempat renang untuk surat tersebut adalah...
berendam. A. Atas perhatian dari pada
Bapak/Ibu saya mengucapkan
Kalimat yang tidak padu pada teks di terima kasih
atas terdapat pada… B. Atas perhatian Bapak/Ibu kami
ucapkan terima kasih banyaK
A. Kalimat 1 C. Atas perhatian  Bapak/Ibu saya
menghaturkan terima kasih
B. Kalimat 2 D. Atas perhatian dan kehadiran
Bapak/Ibu kami  mengucapkan
C. Kalimat 3 terima kasih
E. Atas perhatian  Bapak/Ibu saya
D. Kalimat 4 mengucapkan terima kasih
14. Atas perhatian dan budi baik bapak,
E. Kalimat 5 saya menghaturkan banyak terima
kasih. Perbaikan yang tepat untuk
12. Cermatilah penutup surat lamaran penutupan surat di atas adalah...
pekerjaan berikut! A. Atas perhatian Bapak saya
Besar harapan saya dapat diterima di mengucapkan terima kasih
perusahaan yang Bapak pimpin. Atas B. Atas budi baik  Bapak,saya
kebaikannya, dihaturkan terima kasih. sampaikan  ucapkan terima kasih
C. Atas diterimanya saya bekerja
diperusahaan bapak, saya
ucapkan terima kasih
D. Demikian surat lamaran saya,dan
untuk itu saya sampaikan terima
kasih
E. Dengan diterimanya saya
bekerja,saya mengucapkan terima
kasih

15. Cermati kalimat penutup surat lamaran


pekerjaan di bawah ini!

“Atas perhatiannya, saya ucapkan


terima kasih.”

Penulisan penutup surat tersebut tidak


tepat karena…

A. Penggunakan tanda koma (,) tidak


tepat.
B. Akhiran -nya merujuk pada makna
memiliki.
C. Penulisan terima kasih seharusnya
digabung.
D. Penggunaan kata ganti -nya tidak
tepat, seharusnya menggunakan
kata sapaan.
E. Kata mengucapkan seharusnya
diganti ‘haturkan’.
PAKET 5 4. Labirin

1. Keringat membanjiri tubuhnya .... Tokoh kartun favoritmu


Kata membanjiri pada kalimat tersebut Dan warna kegemaran mu
bermakna ... Kutelusuri di titik mana kitakan bertemu
A. Memenuhi
B. Meningkat Biusaku
C. Membasahi Bius aku dengan tatapanmu, tatapanmu
D. Banyak Misterimu
E. Bercucuran Menyiksaku tapi sungguh candu sungguh
candu
2. Makna asosiasi dapat ditemukan dalam
kalimat-kalimat berikut, kecuali .... Kata candu dalam lagu TULUS tersebut
A. Mereka sering mengolok-oloknya bermakna sebagai ....
karena berasal dari ragunan A. Pohon besar, tingginya 4,5 m, kyunya
B. Beberapa saksi mata mengatakan rapuh
bahwa pada hari-hari tertentu di sekitar B. Getah kering pahit berwarna coklat
pohon itu, sering terlihat putih-putih kekuning-kuningan yang diambil dari
yang melintas dengan cepat. buah Papaver Somniverum.
C. Mulut gang itu telah ditutup pagar kawat C. Cairan kental berwarna hitam yang
berduri agar para perusuh tidak dapat keluar dari rokok yang diisap yang
masuk ke perkampungannya. melekat pada pipa.
D. Sejak 1976,warung pak kumis memang D. Zat narkotika
telah digandrungi orang. E. Sesuatu yan menjadi kegemaran
E. Dengan khidmat, semua peserta
upacara memberi hormat kepada Sang 5. Setelah sekian lama mengabdi di
Merah Putih perusahaan tersebut. Pak Rasak di PHK
oleh direkturnya dengan alasan yang belum
3. Tatap jelas.sehingga dia menuntut haknya dan
mengajukan gugatan ke pengadilan.
Kemanapun langkahmu berderap
Disana ada rasa yang kau kecap PHK disini berarti sebagai ....
Lewat mata sinis pengundang gerimis A. Kabur
Atau teduh yang menghilangkan keluh B. Diturunkan dari jabatan
C. Dipecat
Dari mata-mata yang berbicara tanpa kata D. Dipulangkan
Perlulah kau hadapi dengan cinta E. Dikurangi gajinya

6. Bacalah teks beriukt!


Makna kata berderap dalam larik puisi
tersebut adalah.......
A. Menuju “Indonesia harus membangun
B. Mengayun kemandirian dengan kekuatan yang
C. Beranjak pergi dimilikinya sendiri. Di tengah situasi
D. Meninggalkan tempat ekonomi yang tidak menggembirakan,
E. Berlari Bangsa Indonesia sudah tidak bisa
mengandalkan negara lain untuk
membangun kemandirian. Untuk itu, sumbangan secara sukarela. (12) Pada
Indonesia dapat menempuhnya saat ini, lenong mulai dipertunjukkan atas
dengan meningkatkan daya saing. permintaan pelanggan dalam acara-acara
Caranya, kemampuan sumber daya di panggung hajatan, seperti resepsi
manusia Indonesia dalam hal pernikahan. (13) Selain kedua seni
penguasaan ilmu pengetahuan dan pertunjukan tersebut, budaya arsitektur dan
teknologi harus ditingkatkan”. pertanian Betawi juga mengalami
kemunduran. (14) Contohnya rumah-rumah
Kata itu  pada paragraf tersebut merujuk asli Betawi di kawasan Condet saat ini
pada____ terabaikan. (15) Tambahan lagi, rumah-
A. Tidak bisa mengandalkan negara lain rumah baru orang Betawi saat ini tidak
B. Menghadapi situasi ekonomi global menggunakan gaya arsitektur Betawi.
C. Membangun kemandirian Indonesia
D. Situasi ekonomi global tidak Kata tersebut pada kalimat (11) merujuk
menyenangkan. pada …
E. Meningkatkan Daya saing A. jenis musik tradisional Betawi yang
dipengaruhi suku Tionghoa
7.    (1) Saat ini makin banyak B. seni pertunjukan untuk menyambut
kebudayaan Betawi yang tidak dapat tamu-tamu terhormat
dinikmati masyarakat globalisasi. (2) C. gaya arsitektur rumah Betawi yang
Contohnya adalah seni pertunjukkan ondel- terdapat di Condet
ondel. (3) Seni pertunjukkan ondel-ondel D. pertunjukan dua boneka besar untuk
adalah seni pertunjukkan dengan boneka menolak bala
besar setinggi dua meter. (4) Pada E. teater tradisional Betawi dengan
awalnya, boneka ini berfungsi sebagai gambang kromong
penolak bala dari gangguan roh halus. (5)
Dalam perkembangan selanjutnya,
pertunjukan ondel-ondel juga digunakan 8. (1) Dalam beberapa tahun terakhir,
untuk menambah semarak pesta-pesta mulai banyak pasien anak dan remaja yang
rakyat, peresmian gedung baru, atau memiliki keluhan pada tulang belakang. (2)
menyambut tamu terhormat. Padahal, rasa sakit seperti ini biasanya
dialami orang yang berusia 40 tahun ke
  (6) Contoh lain kebudayaan Betawi atas. (3) Anak dan remaja ini ternyata
yang  terancam tergerus globalisasi adalah gemar menggunakan gadget, seperti
Lenong. (7) Lenong merupakan teater telepon pintar, komputer, dan tablet. (4)
tradisional Betawi yang diiringi musik Banyak dari mereka yang
gambang kromong. (8) Gambang kromong bermain gadget sambil tengkurap,
adalah musik tradisional Betawi yang membungkuk, atau lehernya ke bawah
mendapat pengaruh dari suku Tionghoa. (9) untuk menatap layar monitor, sehingga
Lakon atau scenario lenong umumnya kepala membebani leher. (5) Ada pula yang
mengandung pesan moral, seperti membela badannya bersandar di kursi dengan posisi
kaum lemah. (10) Pertunjukan itu biasanya layar lebih tinggi dari pada mata. (6) Di
diadakan di lapangan terbuka tanpa samping itu, juga ada yang memiringkan
panggung. (11) Ketika pertunjukan tersebut kepala ke satu sisi untuk menjepit gadget di
berlangsung, salah seorang aktor atau antara telinga dan pundak ketika
aktris mengitari penonton untuk meminta menelepon. (7) Apabila ini dilakukan terus
menerus, tulang belakang akan “protes” psikomotor anak akan terbentuk
dengan mengirimkan sinyal nyeri. melalui bangku sekolah.
Kalimat yang menggunakan kata
    (8) Ada berbagai macam sensasi nonformal dalam paragraf di atas adalah
nyeri pada tulang belakang, seperti ditusuk-
tusuk, kesemutan, tersetrum, dan A. (1)
nyeri cenat cenut seperti sakit gigi. (9) Pada B. (3)
nyeri yang diakibatkan gadget, biasanya C. (4)
sensasinya seperti tersetrum. (10) Nyeri ini D. (5)
memang tak berbahaya. (11) Tetapi, jika E. (6)
dibiarkan terus menerus, nyeri ini bisa
merusak postur tulang. (12) Ada tiga 10. Kalimat di bawah ini merupakan
tahapan nyeri pada tulang belakang, yaitu kalimat yang tidak menggunakan
rasa nyeri yang dialami otot, kemudian ragam baku, kecuali ....
menjalar ke sendi, dan terakhir mengenai A. Pada Bahasa Indonesia yang baik dan
tulang. benar membicarakan ejaan dan ragam.
B. Bagi pegawai Bank tidak boleh ikut
Kata ini pada kalimat (7) di atas merujuk lomba itu.
pada .... C. Rencana undang-undang disyahkan
A. posisi menjepit gadget oleh DPR.
B. posisi tengkurap dan membungkuk D. Buah ini bermanfaat untuk kesehatan
C. posisi bersandar di kursi tubuh.
D. posisi menggunakan gadget E. Dalam rapat itu membicarakan masalah
E. posisi menatap layar gadget penurunan prestasi siswa.

11. Kalimat di bawah ini merupakan contoh


9. (1) Salah satu alasan para orang tua kalimat baku, kecuali ....
tidak memberikan pendidikan A. Dalam pertemuan ini kita akan
prasekolah bagi anaknya adalah membicarakan masalah yang terjadi
karena tidak ingin anaknya kemarin.
kehilangan masa kecil. (2) Telah B. Akan kita diskusikan lagi masalah ini
umum dikatakan bahwa masa kecil besok!
adalah masa permainan. (3) Padahal, C. Mereka akan bicarakan lagi masalah
pendidikan prasekolah anak bukanlah itu.
pendidikan yang "menyeramkan" D. Setiap murid harus dapat mengerjakan
seperti halnya sekolah yang tugas itu dengan baik.
menuntut anak untuk ini dan itu. (4) E. Marilah kita memulai diksusi ini!
Pendidikan anak usia dini tetap
memberikan kebebasan kepada anak 12. Di bawah ini yang menggunakan
untuk bermain dalam konteks ragam bahasa baku adalah ...
bermain sambil belajar. (5) Lebih A. Semua pegawai daripada pabrik ini
daripada itu, pendidikan anak usia sudah pulang.
dini penting karena dapat membentuk B. Kami mengucapkan terima kasih atas
kesiapan diri anak dalam kehadiran saudara.
menghadapi masa sekolah. (6) C. Udah deh, jangan mengganggu terus!
Kecerdasan kognitif, afektif, dan D. Bilang dulu sama saya punya ibu.
E. Kami menghaturkan terima kasih atas Kalimat di atas akan menjadi kalimat baku
perhatiannya. jika diperbaiki dengan cara berikut,
kecuali ....

13. Dengan perkembangan teknologi telah A. Mengganti kata dua ratus dengan 200.
menuntut para pendidik untuk mencari B. Mengganti kata muncul dengan terbit
metode mengajar yang lebih menarik. C. Mengganti kata ialah dengan yaitu
Kalimat di atas tidak baku dan dapat D. Mengganti kata tetapi dengan namun
diperbaiki dengan cara .... E. Mengganti kata menggunakan dengan
mengenakan
A. Menghilangkan kata telah
B. Mengubah kata menuntut menjadi
dituntut
C. Menghilangkan kata dengan
D. Meletakkan dengan perkembangan
teknologi di akhir kalimat
E. Meletakkan para pendidik di awal
kalimat

14. Para mahasiswa termasuk mahasiswa


Program Pasca Sarjana merupakan
civitas academica sehingga memiliki
hak yang sama untuk menggunakan
fasilitas perpustakaan.
Kalimat di atas akan menjadi kalimat baku
jika ejaannya diperbaiki sebagai berikut,
kecuali ....

A. Memberi tanda koma setelah para


mahasiswa
B. Penulisan Pasca Sarjana dirangkaikan
C. Setelah kata sarjana diikuti tanda koma
D. Kata sehingge didahului tanda koma
E. Kata Program Pasca Sarjana
semuanya ditulis dengan huruf kecil

15. Pukul 08.00 di Tomok, Pulau Samosir,


matahari baru saja muncul di Danau
Toba. Udara dingin dan kabut belum
beranjak. Tetapi, sekitar dua ratus
warga desa telah bergerak ke rumah
peninggalan Raja Sidabutar dengan
menggunakan pakaian adat Batak,
ialah kain ulos dan ikat kepala tiga
warna: merah, putih, dan hitam.
PAKET 6 a. Prof. Dr. Surya Sumantri SE.
b. Prof. Dr. Surya Sumantri, S.E.
1. Sejak kepergian Ibunya Andri tidak pernah c. Prof. Dr Surya Sumantri S.E.
lagi ke rumah. Andri memilih tinggal di asrama d. Prof. Dr. Surya Sumantri, SE
bersama dengan teman-temannya, dari yang e. Prof Dr. Surya Sumantri, S.E
tua hingga yang muda. Olehnya itu, banyak 5. Contoh penulisan kelompok kata yang benar
orang bersedih memikirkan nasibnya. Sejak di bawah ini kecuali...
peristiwa itu banyak perubahan yang terjadi
a. Dari pada
dalam diri Andri. Dia memilih meninggalkan
b. Menindaklanjuti
semuanya dari pada tinggal di rumahnya yang c. Menanda tangani
penuh kenangan bersama ibunya. Sesekali dia d. Bertanda tangan
hanya mengunjungi dan bersilaturahmi kepada e. Dilipat gandakan
tetangga dekatnya.
6. Penulisan kata baku berimbuhan yang tepat
Penulisan kata depan bercetak miring yang tidak pada kata yang digari bawahi terdapat dalam
tepat terdapat pada kata... kalimat...
a. Kami sudah mengetrapkan peraturan
a. dari pada itu sebagaimana mestinya.
b. di asrama
c. di rumahnya b. Kami sudah menterapkan peraturan itu
d. ke rumah sebagaimana mestinya.
e. kepada c. Kami sudah menerapkan peraturan itu
2. Penulisan partikel yang benar di bawah ini sebagaimana mestinya.
adalah.... d. Kami sudah menyeterapkan peraturan
a. Bacalah buku itu dengan benar! itu sebagaimana mestinya.
e. Kami sudah terapkan pertauran itu
b. Apa lah gunanya bersedih hati?
sebagaimana mestinya.
c. Siapa kah wanita yang menawan itu?
7. Penulisan deretan kata yang benar berikut ini
d. Apapun itu aku sanggup. adalah...
e. Sudah (kah) anda istirahat? a. Atlit, apotik, aktifitas
3. Penulisan nama orang yang tepat di bawah b. Managmen, khabar, fikir
ini adalah....
c. Karier, aerobik, kuitansi
a. A.H Nasution
d. Analisa, metode, diagnosa
b. Suman H.S
e. Kwalitas, kwantitas, kwitansi
c. WR. Supratman
8. Perhatikan paragraf berikut!
d. MBA Yusuf Akbar
Untuk menuju ... pantai Parangtritis
e. HJ Sumarni yang terletak 27 kilometer .... kota Yogyakarta,
4. Penulisan nama gelar yang benar adalah... kita dapat menempu salah satu .... dua jalur
jalan. Jalur pertama lurus... arah Selatan d. Mahasiswa semester enam akan
Yogyakarta hingga sampai .... Kretek. Jalur melakukan demonstrasi.
kedua jalan yang melintasi Imogiri.
e. Pada tahun ke enam mahasiswa itu baru
Kata depan yang tepat untuk melengkapi dapat menyelesaikan studinya.
paragraf tersebut adalah...

a. Ke, di, dari, ke


b. Ke, dan, dari, di, ke
c. Ke, dari, dari, ke, di
d. Ke, ke, dari, dari, di
e. Ke, dan, ke, dan, di

9. Pada dunia sastra ada beberapa jenis keritik


syang digunakan oleh para kritikus dalam
membedah suatu karya, namun selain
menggunakan keritik orang-rang biasanya
cenderung menganalisis karya sastra tersebut
dengan menggunakan dua bentuk analisis, yaitu
intrinsik dan extrinsik. Intrinsik menganalisa
karya berdasarkan bentuk dan gayanya secara
komplek. Sedangkan extrinsik menganalisis
karya sasta berdasarkan nilai-nilainya.

Penulisan kata serapan pada kutipan di atas


tidak sesuai PUEBI. Perbaikan penulisan kata
serapan yang sesuai PUEBI adalah...

a. Kritik, ekstrinsik, komlek


b. Kritiks, ekstrinsik, kompleks
c. Kritis, ektrinsik, komplek
d. Kritik, ekstrinsik, kompleks
e. Kritis, ektrinsik, komplek

10. Penulisan angka atau lambang bilangan


yang benar terdapat dalam kalimat....

a. Ulang tahun kemerdekaan RI yang ke


50 diperingati secara meriah.

b. Sri Sultan Hamengku Buwono ke X


dikenal sebagai tokoh yang dekat dengan
rakyat kecil.

c. Petenis kita harus puas memperoleh


juara ketiga saja.
PAKET 7 a. Dan bertanya, :Siapa gadis itu?”.
b. Rok “canda” sudah tidak popular
1. Cermati penulisan judul di bawah ini!
lagi di Indonesia.
Buku adalah jendela dunia, sudahkah anda
c. Lana tinggal di Jalan Hakim,
membaca “Korelasi membaca dengan tingkat
kecerdasan intelektual dan hubungan sosial?” Medan.
d. Rate of inflation, ‘LAU INFLASI’, di
Perbaikan penulisan judul karangan di atas yang
tepat adalah... Indonesia mulai menurun.
e. Rina menjaurai perlombaan itu,
a. Korelasi Membaca Dengan Tingkat
Kecerdasan Intelektual dan hubungan karena rajin belajar.
Sosial 4. Perhatikan kalimat berikut ini!
b. Korelasi Membaca Dengan Tingkat
Kecerdasan Intelektual Dan Hubungan PT Pertamina Gas (Pertagas) (...) anak
Sosial usaha PT Pertamina di sektor gas bumi (...)
c. Korelasi Membaca dengan Tingkat mencatatkan laba bersih hingga Agustus 2017
Kecerdasan Intelektual dan Hubungan sebesar 90,6 juta dolar AS.
Sosial
d. Korelasi Membaca dengan Tingkat Tanda baca yang tepat untuk melengkapi
Kecerdasan Intelektual Dan Hubungan kalimat tersebut adalah...
Sosial a. Tanda baca titik dua (:) sebelum kata
e. Korelasi Membaca Dengan Tingkat
Kecerdasan Intelektual dan Hubungan anak dan tanda baca koma (.) setelah
Sosial kata bumi.
b. Tanda baca titik (.) sebelum kata anak
2. Pemakaian tanda baca yang benar di dan tanda baca titik dua (: ) setelah
bawah ini terdapat pada kalimat... kata bumi.
a. Karena sedih, adik menangis di c. Tanda baca koma (,) sebelum kata
kamarnya. anak dan tanda baca koma (,) setelah
b. Adik dan kaka belum pulang, kata bumi.
walaupun sudah malam. d. Tanda baca hubung (-) sebelum kata
c. Ibu membeli nasi, jus, pisang dan anak dan tanda baca pisah (-) setelah
jeruk. kata bumi.
d. Murid, dan guru sedang mengikuti e. Tanda baca koma (,) sebelum kata
pacara bendera. anak dan tanda baca hubung (-)
e. Ban mobil kami pecah, karena itu setelah kata bumi.
kami tidak bisa datang.

5. Penulisan catatan kaki yang tepat di


3. Penggunaan tanda baca yang tepat bawah ini adalah....
pada kalimat di bawah ini adalah...
a. Alisjahbana, S. Takdir, Tata Bahasa
Baru Bahasa Indonesia. Jilid 2 7. a) Kemerdekaan itu ___ hak segala
(Jakarta: Pustaka Rakyat, 1950), hlm. bangsa __ harus dipertahankan.
25. b) Kemerdekaan itu ___ hak___ segala
b. Hilman, Hadikusuma, Ensiklopedi bangsa___ harus dipertahankan.
c) Kemerdekaan itu hak ___ segala
Hukum Adat Dan Adat Budaya bangsa ____ harus dipertahankan.
Indonesia (Bandung; Alumni, 1977) d) Kemerdekaan ___ itu___ hak segala
hlm.12. bangsa ____ harus dipisahkan.
e) Kemerdekaan itu____ hak segala
c. Poerwadarminta, W.J.S. bahasa
bangsa harus ____dipertahankan.
indonesia untuk karang-mengarang
Penulisan tanda baca pisah (____) yang tepat
(Jogjakarta: UP Indonesia, 1967),
terdapat pada nomor....
hlm.4.
a. 1
d. Gunawan, Ilham. Kamus Politik
b. 2
Internasional (Jakarta: Restu Agung.
c. 3
e. Junus, H. Mahmud. 1973. Kamus
d. 4
Arab-Indonesia. Jakarta: Yayasan
e. 5
Penyelenggara Penerjemah/Penafsir
Alquran.
8. Perhatikan paragraf berikut!
1) Mabel adalah salah satu dari tujuh
6. Penulisan perihal surat yang benar
anak dari Pearl dan Edward
adalah...
Treadwell. 2) Ayahnya merupakan
a. Perihal: Permohonan Bantuan Tenaga
seorang pelayan yang meninggal
Pengajar Bahasa Indonesia dan
karena penyakit jantung ketika dia
Sejarah.
masih kecil. Pada usia 10 tahun,
b. Perihal: Permohonan Bantuan tenaga
dia menderita peritonitis, penyakit
pengajar bahasa Indonesia dan
yang mengancam jiwa akibat
Sejarah.
peradangan rongga perut yang
c. Perihal : Permohonan bantuan tenaga
menyebabkan infeksi dapat
pengajar bahasa Indonesia dan
menyebar ke seluruh tubuh. 3)
sejarah.
Keluarga tidak mampu membayar
d. Perihal: Permohonan Bantuan Tenaga
biaya operasi sehingga dia tidak
Pengajar Bhs. Indonesia dan Sejarah.
pernah menerima perawatan.4)
e. Perihal; Permohonan Bantuan Tenaga
Mabel pada akhirnya dpat sembuh
Pengajar Bahasa Indonesia dan
namun keparahan penyakitnya
Sejarah.
kelak membuatnya tidak subur. 5)
Dia lulus di Ohio State University b. 2
dan menjadi jurnalis. Kemudian, c. 3
dia bekerja di departeman perang. d. 1 dan 2
6) Di sana, dia mendorong e. 2 dan 3
penghapusan perbedaan di antara
prajurit yang dimakamkan di 10. Penggunaan tanda penghung yang
Pemakaman Nasional Arlington. 7) tidak tepat pada kalimat di bawah ini
Dia menilai sangat tidak adil untuk adalah....
dibeda-bedakan dalam kematian a. Berulan-ulang
setelah, melayani negara. (http//: b. Peringkat ke-2
harian-kompas-berita- c. Se-indonesia
internasional) d. Ciptaan-Nya
e. Pen-tacklean
Penggunaan tanda baca yang tidak tepat
terdapat pada kalimat nomor .... Jawaban E (Tanda penghubung dipakai untuk
merangkai unsur bahasa Indonesia dengan
a. 1
unsur asing, penulisan yang tepat untuk pilihan
b. 2 E adalah Pen-tackle-an).
c. 3 dan 4
d. 5 dan 6
e. 7

9. Perhatikan paragram di bawah ini!

Rayonisasi dulu dan sekarang itu berbeda.


Kalau dulu rayonisasi mengelompokkan anak
untuk melanjutkan sekolah pada daerah
tertentu, sekarang hanya membantu orang tua
murid mendaftar sekolah. Jadi lebih bersifat
pelayanan. "Sekarang, murid bebas memilih
sekolah", kata Abdul Rochim, Kepala Subdinas
Pendidikan Sekolah Lanjutan Tingkat Pertama
(SLTP) Dinas Pendidikan Dasar Provinsi DKI
Jakarta, saat ditemui Kompas, sebelum acara
Sosialisasi PSB 2002-2003 di Jakarta, Jumat
(10/5). (Sumber: Kompas, Sabtu, 11 Mei 2002).

Penggunaan tanda baca koma yang tidak


tepat terapat pada kalimat....

a. 1
PAKET 8 Barat. “Jika ingin makan telur penyu, datanglah
ke Pantai Muaro Padang,” begitu tulis

panduan tersebut.
1. Cermati paragraf berikut!       
Telur-telur penyu tersebut dipasok dari Pulau
(1) Udara yang dihirup makhluk hidup Penyu di pesisir selatan dan pulau-pulau
mempunyai kadar oksigen 21 persen. (2) Kadar
tersebut harus tetap terpelihara. (3) Namun kecil lainnya di sekitar Padang. Bila pasokan
setiap tahun, dunia industri akan menghabiskan kurang karena banyaknya permintaan, telur
kira-kira 41 persen oksigen untuk penyu
menggerakkan roda perindustriannya. (4) Kalau
dituliskan dalam bilangan nyata, oksigen didatangkan dari provinsi tetangga, seperti Riau.
dihabiskan oleh industri kurang lebih 400 ton Telur penyu dianggap memiliki khasiat untuk
setiap menit yang akan diambil dari udara tanpa obat awet muda, obat kuat, dan kebugaran.
pengembalian. (5) Meskipun bilangan itu cukup
besar, keadaan itu belum sampai membuat kita (Tempo, 17 – 23 November 2008)
sesak nafas.   
Pernyataan-pernyataan di bawah ini merupakan
Kalimat fakta dalam paragraf tersebut terdapat fakta dari teks di atas kecuali ....
pada kalimat nomor . . .

A. (1)
A. Pantai Muaro Padang menjadi pasar
B. (2) penjualan telur penyu di Sumatera Barat.

C. (3) B. Penjualan telur penyu di Pantai Muaro


Padang masuk di dalam panduan wisata
D. (4)
kuliner.
E. (5)
C. Panduan wisata kuliner di Padang
dikeluarkan oleh Dinas Pariwisata Sumatera
2. Perhatikan teks berikut! Barat.
Pantai Muaro Padang kini memang menjadi D. Telur penyu di pasok dari pulau-pulau kecil di
pasar regional penjualan telur penyu. Jikadi sekitar Padang dan Pulau Penyu.
daerah lain orang harus sembunyi-sembunyi
berjualan telur penyu, di Pantai Muaro ini E. Telur penyu mempunyai khasiat obat awet
muda, obat kuat, dan kebugaran.
penjualan telur penyu tersebut sangat terbuka,
sangat bebas. Bahkan Pantai Muaro Padang

yang berjarak hanya 1 km dari kantor Wali Kota 3. Perhatikan teks berikut
Padang menjadi maskot wisata setempat
Indonesia memiliki pulau yang jumlahnya
karena masuk dalm panduan wisata kuliner puluhan ribu. Sebagian besar pulau tersebut
yang dikeluarkan Dinas Pariwisata Sumatera telah
diberi nama dan berpenghuni. Pemberian nama 5. Bacalah teks berikut!
pulau dilakukan agar mudah diidentifi kasi.
(1)Tenaga listrik pertama kali ditemukan oleh
Pemerintah sekarang ini sedang menggalakkan Thomas Alva Edison pada tahun 1879. (2) Saat
program pemberian nama pulau terluar di itu, dia berhasil menemukan bola lampu pijar
yang sangat terkenal sampai ke seluruh dunia.
Indonesia. Pemberian dan pemasangan nama (3) Saat itu Thomas Alva Edison masih berusia
pulau dilakukan oleh Departemen Kelautan 17 tahun. (4) Penemuan listrik tersebut sangat
terasa manfaatnya oleh semua negara termasuk
yang melibatkan TNI Angkatan Laut.
Indonesia. (5) Jika hal itu tidak terjadi mungkin
Berdasarkan teks tersebut pernyataan dalam di Indonesia listrik tidak masuk ke pelosok desa.
pilihan tersebut yang merupakan opini Kalimat berisi fakta pada paragraf tersebut
ditandai nomor ....
adalah ... (SNMPTN 2009)
A. (1) dan (3)
A. Indonesia banyak memiliki pulau yang belum
diberi nama. B. (2) dan (4)

B. Pulau yang belum diberi nama perlu C. (2) dan (5)


dilakukan pemberian nama.
D. (4) dan (5)
C. TNI Angkatan Laut sedang melakukan
E. (1) dan (5)
pemasangan nama pulau.

D. Program pemberian nama pulau sedang


6. Perhatikan teks berikut!
dilakukan pemerintah.
(1) Memberikan “asupan” makanan yang sehat
E. Departemen Kelautan bekerja sama dengan
edan seimbang adalah cara yang baik untuk
TNI Angkatan Laut.
memastikan tumbuh kembang serta menjaga
tubuh tetap sehat dan kuat. (2) Pola makan
4. Perhatikan teks berikut yang dianjurkan adalah yang mengandung tiga
kelompok zat nutrisi, yaitu sumber zat tenaga,
(1) Udara di Bogor terasa dingin. (2) Kali ini pembangun, dan pengatur. (3) Makanan yang
dinginnya melebihi hari-hari sebelumnya. (3) tergolong dalam sumber zat tenaga adalah nasi,
Dinginnya suhu udara di Bogor mencapai 24ºC. tepung-tepungan, umbi-umbian, gula dan
(4) Data tingkat suhu udara ini, terdapat di minyak. (4) Sumber zat pembangun terdapat
papan informasi pengukur suhu di jalan-jalan pada makanan seperti ikan, telur, kacang-
besar di kota Bogor. Dua kalimat pendapat pada kacangan. (5) Sedang sumber zat pengatur
teks tersebut ditandai dengan nomor .... banyak terdapat pada sayur dan buah, terutama
yang berwarna hijau dankuning. Kalimat yang
A. (1) dan (2)
berisikan fakta pada paragraph tersbut adalah . .
C. (2) dan (3) .

B. (1) dan (3) A. (1), (2), dan (3)

D. (2) dan (4) B. (1), (3), dan (4)

E. (3) dan (4) C. (2), (3), dan (4)


D. (3), (4), dan (5) Kata penghubung yang tepat untuk melengkapi
bagian yamg rumpang dalam paragraf tersebut
E. (1), (4), dan (5) adalah...

A. atau, tetapi, sebab


7. Perhatikan teks berikut B. yaitu, bahkan, bahwa
C. serta, yakni, dan
Tindak konsiten dan berkelanjutan merupakan
D. dan, serta, jika
tantangan kita. Baru saja dipertontonkan lagi
E. karena, misalnya, apabila
kelemahan kita pada bencana gempa Bengkulu
dan sumatera Barat. Bantuan segera diangkat
9. Perhatikan teks berikut!
dengan pesawat terbang ke Bengkulu dan
padang. Segera pula tampak lewat laporan tujuan pelatihan kecerdasan emosional ini ...
multimedia keluh kesah warga luar ibu kota orang tua sadar terhadap emosi mereka sendiri
provinsi yang belum mendapatkan bantuan. dan dapat menggunakan kepekaan mereka ...
Setelah dua minggu pun keluhan itu masih menyelaraskan diri secara tulus dengan
terdengar dan tampak. perasaan anak mereka.
Opini pada tajuk tersebut rencana tersbut Kata penghubung yang tepat untuk melengkapi
adalah . . . bagian yamg rumpang dalam paragraf tersebut
adalah...
A. penanganan bencana di negara kita masih
lemah A. demi, bagi
B. bila, agar
B. bantuan harus segera dikirimkan kepada
C. agar, untuk
korban bencana alam
D. kalau, demi
C. pemerintah memberikan bantuan kepada E. bahwa, yang
korban bencan secara merata
10. Bacalah teks tersebut!
D. laporan multimedia dapat membantu dalam
penanganan bencana Berbagai cara akan ditempuh oleh siswa yang
baru lulu SMA untuk dapat berebut kursi di
E. Bencana gempa melanda Bengkulu dan perguruan tinggi favoritnya. Ada siswabelajar
Sumatra Barat keras sampai begadang ... ada yang mengikuti
berbagai bimbingan tes. Disamping itu, ada
yang melalui jalur khusus, yaitu pemberian
8. Perhatikan teks berikut kesempatan kepada siswa yang mampu secara
finansial untuk duduk di universitas pilihannya.
Jasa-jasa ekologi yang terdapat di Kawasan Ada juga yang masuk universitas tanpa tes ...
Ekosistem Leuser mencakup penyediaan air dia memiliki prestasi gemilang ... memiliki nilai
bersih, pengendalian erosi ... banjir, pengaturan akademis yang ditargetkan suatu perguruan
iklim lokal, penyerapan karbon, perikanan air dengan melalui jalur PMDK, dan ada jalur
tawar, ... keindahan alam mendukung industri pemberian beasiswa bagi siswa yang
pariwisata. Jasa-jasa ini hanya dapat tersedia ... berprestasi.
Kawasan Ekosistem Leuser dijaga dan
dipelihara fungsinya sebagai suatu kesatuan Kata penghubung yang tepat untuk melengkapi
interaksi yang utuh. bagian yamg rumpang dalam paragraf tersebut
adalah...
A. dan, karena, atau Paket kedua mengambil tema prospekekonomi
B. dan, tetapi, atau rakyat
C. hingga, karena, tetapi
D. hingga, jika, atau Kedua kalimat diatas dapat dijadikan kalimat
E. dan, jika, tetapi majemuk dengan konjungsi

A. dan
11. Perhatikan teks berikut! B. bahkan
Campur semua bahan hingga merata dan C. padahal
simpan dalam bejana kaca yang tertutup D. sedangkan
rapat ... ambil 1/5 bagian scrub. Kemudian E. tetapi
masukkan kedalam sebuah mangkuk dangkal,
campur bahan dengan minyak almond sampai 14. (1) Kemarahan dapat meningkatkan
menjadi semacam pasta. Oleskan scrub tadi peluang terjadinya serangan jantung.
diseluruh tubuh sambil dipijat dengan lembut. (2) Hal ini disebabkan oleh pelepasan
Berikan perhatian khusus pada area kulit hormon stres akibat marah. (3) ... ,
kering ... keras seperti sikut, lutut, dan tumit. kemarahan membuat sel-sel otot
Bersihkan sisa-sisa scrub ... berendam atau jantung membutuhkan banyak oksigen
dan menyebabkan kekentalan pada
menyiram.
darah meningkat. (4) Keadaan tersebut
Kata penghubung yang tepat untuk melengkapi membuat detak jantung meningkat
melebihi batas wajar dan
bagian yamg rumpang dalam paragraf tersebut
menyebabkan naiknya tekanan darah.
adalah... Kata penghubung yang tepat untuk
mengisi rumpang pada kalimat ketiga di
A. setelah itu, dan, sebelum
atas adalah ...
B. sementara itu, atau, sehingga
C. selain itu, serta, jika
a. akibatnya (D) dan
D. sebab itu, atau, supaya
E. karena itu, juga, bila b. selain itu (E) akhirnya
c. sebab
12. Musik di Insonesia sangat beragam ...
banyaknya jumlah suku di Indonesia.

Kata penghubung yang tepat untuk melengkapi


bagian yamg rumpang dalam paragraf tersebut
adalah...

A. dan
B. walaupun
C. sehingga
D. karena
E. juga

13. perhatikan kalimat berikut !

Paket pertama mengambil tema Puasa dan


Krisis Moral.
PAKET 9 A. Liburan saat lebaran sangat
mengasyikan
1. Hal yang bukan ciri-ciri sebuah B. Mari kita berlibur ke Sari Ater ,
teks iklan adalah.... Subang
A. Disusun secara subjektif dan C. Di Ciater, Subang , banyak tempat
hoax     wisata yang mengasyikan
B. Ditulis singkat, padat, jelas, dan D. Berpetualang di tempat wisata
mudah dipahami sungguh mengasyikan
C. Tidak menyingguang pihak lain E. Tidak ada tempat yang semenarik
D. Menarik perhatian banyak orang Ciater, Subang
E. Bersifat informatif dan komunikatif
7. Kalimat iklan yang berhubungan
2. Hal yang bukan termasuk unsur iklan dengan masalah kependudukan
adalah.... adalah...
A. Sumber A. Beli satu dapat dua
B. Gambar B. Wajib belajar sembilan tahun
C. Pesan C. Dengan PIN anak Indonesia bebas
D. Media polio
E. Sasaran/penerima D. Dua anak lebih baik
E. Kemana Anda setelah tamat
3. Berikut ini yang bukan fungsi iklan sekolah?
adalah...
A. Memberikan layanan 8. Perhatikan ilustrasi berikut!
B. Memberikan informasi
C. Menceritakan peristiwa OSIS SMP Tunas
D. Memberikan penjelasan mengadakan Lomba Marawis.
E. Memperluas jangkauan Tujuannya ingin meningkatkan
pemasaran
keimanan terhadap Tuhan Yang Maha
4. Makna istilah ‘storyboard’ dalam Iklan Esa.
adalah.... Slogan yang sesuai dengan ilustrasi
A. Iklan media cetak tersebut adalah ...
B. Rangkaian berbagai iklan A. Lomba Marawis tingkatkan rasa
C. Iklan media elektronik kesenian.
D. Rangkaian gambar yang B. Dengan Lomba Marawis, kita
membentuk sebuah cerita
tingkatkan iman.
E. Rangkaian penjelasan iklan
C. Dengan lomba Marawis kenalkan
5. Ciri-ciri bahasa iklan sebagai budaya daerah.
berikut, kecuali... D. Lomba Marawis pertebal rasa
A. Menggunakan pilihan kata tepat, budaya kebangsaan.
menarik, logis, dan sopan E. Lomba Marawis Osis SMP.
B. Menggunakan ungkapan atau
majas dan memiliki daya sugesti
9. Perhatikan ciri-ciri berikut ini!
C. Disusun untuk menonjolkan
informasi yang dipentingkan a. Bersifat persuasife
D. Harus tepat menuju sasaran iklan b. Mengutamakan unsur kata-kata
E. Menggunakan bahasa daerah c. Mengutamakan unsur gambar,
kata, gerak, dan suara.
6. Kalimat persuasif yang sesuai dengan d. Mengutamakan kekuatan gambar
iklan tentang liburan adalah.... dan kata-kata
e. Dipajang di tempat umum masyarakat, para pegawai puskesmas
f. Disampaikan melalui media massa membuat poster agar masyarakat
tertarik untuk datang ke puskesmas
Berdasarkan ciri di atas, yang memeriksakan kesehatan.
merupakan ciri-ciri poster adalah… Kalimat poster yang sesuai dengan
ilustrasi tersebut adalah . . . .
A. a, b, dan c a. Kemarin sehat, hari ini sehat,
B. a, b, dan e besok sehat, lusa sehat.
C. a, c, dan e b. Ayo ke Puskesmas, Jangan Ragu
D. a, d, dan e c. Memberi bantuan pada orang lain
E. a, e, dan f berarti menyelamatkan orang lain.
d. Menggunakan obat harus secara
10. Kalimat poster yang cocok untuk teratur dan sesuai hukum.
mengajak masyarakat untuk tidak e. Aku sehat dan kuat.
meragukan kualitas produk dalam
negeri ialah…
A. Ayo pakai produk dalam negeri
dengan kualitas bagus
B. Kualitas produk dalam negeri
harus dihargai
C. Kambali pakai produk dalam
negeri
14. Poster tersebut adalah jenis poster …
D. Produk dalam negeri, kualitas luar
a. Niaga
negeri
b. Pendidikan
E. Produk dalam negeri tak kalah
c. Pelayanan Kesehatan
saing
d. Kegiatan
e. Kampanye
11. Berikut ini jenis-jenis poster, kecuali…
A. Poster niaga
15. Yang bukan termasuk syarat poster
B. Poster pendidikan
adalah …
C. Poster hiburan
a. Poster sebaiknya dikombinasikan
D. Poster informasi
dengan bentuk gambar.
E. Poster kampanye
b. Poster harus mampu menarik
.
minat khalayak.
12. “Mari Giat Membaca”. Pilihan jawaban
c. Media poster harus
untuk mengubah kalimat poster
mempergunakan bahan yang tidak
tersebut agar lebih menarik adalah…
mudah rusak atau sobek.
A. Mari Budayakan Gemar Membaca
d. Ukuran poster sebaiknya
B. Ayo Kita Rajin Membaca
disesuaikan dengan tempat atau
C. Tiada Hari tanpa membaca
lahan pemasangan serta target
D. Harimu untuk Membaca
pembaca.
E. Ayo Gencarkan Baca Buku
e. Poster bergambar menarik anak-
anak
13. Bacalah ilustrasi berikut!
Dalam rangka membantu dan
mengabdi untuk kesehatan
PAKET 10 A. 12 Nopember 2009
B. Jakarta, 12 November 2009
C. 12 November 2009
D. Jakarta, 12 Nopember 2009
1. Bacalah kutipan surat berikut dengan
E. Jakarta. 12 November 2009
saksama kemudian kerjakan dua soal
di bawahnya!

SMP CEMERLANG

Jalan Bendi Kebayoran Lama Jakarta Selatan 2. Isi surat yang tepat untuk mengisi
bagian rumpang (2) pada surat
______________________________________
tersebut adalah...
________________________

Nomor : 11/S.0301/G/09 `````````````````````````1)


--------- A. Sehubungan dengan pelaksanaan
Lomba Sekolah Sehat Tahun 2009,
Hal : Undangan
kami mengharap izin Bapak /Ibu pada
hari, tanggal : Sabtu , 14 November
2009
Yth. Para Guru dan Karyawan waktu : Pukul 08.00
tempat : Ruang Aula Cemerlang
SMP Cemerlang
acara : Persiapan Lomba
di Jakarta B. Sehubungan dengan pelaksanaan
Lomba Sekolah Sehat Tahun 2009,
kami mengharap kehadiran Bapak /Ibu
pada
Dengan hormat,
hari, tanggal : Sabtu , 14 November
(2) 2009
---------------------------------------------------------------- waktu : Pukul 08.00
----------------------------- tempat : Ruang Aula SMP Cemerlang
acara : Persiapan Lomba
---------------------------------------------------------------- C. Sehubungan dengan pelaksanaan
--------------------------------- Lomba Sekolah Sehat Tahun 2009,
kami mengharap bantuan Bapak /Ibu
----------------------------------------------------------------
pada
---------------------------------
hari, tanggal : Sabtu , 12 November
Demikian surat undangan ini kami sampaikan. 2009
waktu : Pukul 20.00
Atas perhatian Bapak/Ibu kami ucapkan terima tempat : Lapangan SMP Cemerlang
kasih. acara : Evaluasi Hasil Lomba
D. Sehubungan dengan pelaksanaan
Lomba Sekolah Sehat Tahun 2009,
Tanggal surat yang tepat untuk mengisi nomor kami mengharap sambutan Bapak /Ibu
(1) pada surat tersebut adalah .... pada
hari, tanggal : Sabtu , 12 November B. Silakan saja, laksanakan
2009 dengan sebaik-baiknya.
waktu : Pukul 08.00 Koordinasi dengan petugas
tempat :Ruang Aula SMP Cemerlang terkait. Jika telah dilaksanakan
acara : Evaluasi Kegiatan segera melaporkan hasilnya.
E. Sehubungan dengan pelaksanaan C. Jangan ragu-ragu untuk
Lomba Sekolah Sehat Tahun 2009, melaksanakan peringatan
kami mengharap bantuan Bapak /ibu Hardiknas. Laksanakan
pada dengan sebaik-baiknya jika
hari, tanggal : Sabtu , 12 November sudah selesai harap lapor.
2009 D. Saya dukung terus,
waktu : Pukul 20.00 laksanakan peringatan
tempat : Lapangan SMP Cemerlang Hardiknas, koordinasi dengan
acara : Evaluasi Hasil Lomba petugas terkait secepatnya
laksanakan.
E. Saya akan melaksanakan
dengan sebaik-baiknya.
3. Perhatikanlah kutipan surat berikut!

Kami beritahukan bahwa dalam rangka


memperingati hari pendidikan nasional, 4. Perhatikanlah surat undangan berikut!
SMP Harapan Bakti akaln
menyelenggarakan lomba baca puisi tingkat Dengan ini kami mengundang Bapak/
SMP se-provinsi. Kegiatan tersebut akan Ibu untuk menghadiri rapat rutin awal
dilaksanakan pada: tahun pelajaran 2010/2011 yang akan
diselenggarakan pada
hari, tanggal : Senin, 26 s.d. 28 April 2009,
hari, tanggal : Sabtu, 7 Agustus 2010,
pukul : 8.00 s.d. 14.00 WIB,
waktu : pukul 9.00 – 11.00 WIB,
tempat : Aula SMP Harapan Bakti.
tempat : Aula SMP Prapanca, Bogor,
Sehubungan dengan hal tersebut, kami
mengajukan permohonan izin untuk acara : Ketetapan Uang Dana Bulanan
menyelenggarakan kegiatan tersebut. Atas Tahun Pelajaran 2010/2011.
perhatian Bapak, kami ucapkan terima
kasih. ….

Isi balasan yang tepat surat dinas tersebut Kalimat penutup surat undangan
adalah… tersebut yang tepat adalah…

A. Berkaitan dengan A. Atas kehadiran dan doa restu


permohonan izin Saudara Bapak/ Ibu kami ucapkan
dalam rangka memperingati terima kasih.
Hardiknas yang akan B. Atas waktu yang diluangkan,
dilaksanakan pada Senin, 26 kami ucapkan terima kasih.
s.d. 28 April 2009, pada C. Kehadiran Bapak/ Ibu sangat
dasarnya kami setuju, silakan kami harapkan tepat pada
dilaksanakan sebaik-baiknya. waktunya.
D. Mengingat pentingnya acara Jalan Merdeka 12, Tarutung Medan
tersebut, kami mengharapkan
Bapak/ Ibu dapat tepat waktu.
Atas perhatian Bapak/ Ibu, Salam kangen,
kami ucapkan terima kasih.
E. . Atas kehadiran dan doa restu […]
Bapak/ ibu Kami ucapkan
terima kasih. Semoga kamu sekeluarga sehat-sehat
seperti halnya aku dan keluargaku.

5. Perhatikan kutipan alamat surat


berikut! Tidak terasa sudah setengah tahun kita
berpisah, aku benar-benar kehilangan
Kepada sahabat baik, sejak kepindahanmu.

Yth. Bapak Direktur PT Angin Topan

Jalan angkasa baru No.50 Sahabatmu,

Jakarta Timur Inaara

Perbaikan penulisan alamat surat


tersebut yang tepat adalah…
Kalimat yang tepat untuk melengkapi
A. Yth. Direktur PT Angin Topan surat pribadi tersebut adalah ...
Jalan angkasa baru No.50
B. Yth. Direktur PT Angin Topan A. Senang dapat bersilaturahmi
Jalan Angkasa Baru 50 denganmu via surat.
Jakarta Timur B. Kuharap kamu selalu dalam
C. Yth. Direktur PT angin topan keadaan bahagia.
Jalan Angkasa Baru No.50 C. Hai, Anggi, bagaimana
Jakarta Timur kabarmu sekarang?
D. Yth. Direktur pt Angin Topan D. Hai Inaara keluargamu baik-
Jl. Angkasa Baru 50 baik saja bukan?
Jakarta Timur E. Semoga kamu selalu dalam
E. Yth. Direktur PT Angin Topan keadaan baik
Jl. Angkasa Baru 50
Jakarta timur 7. yg bukan termasuk struktur surat
pribadi adalah......
A. tanggal surat
B. pengirim
6. Perhatikan surat berikut! C. salam penutup
Jakarta, 14 Februari 2009 D. nomor surat
E. kepala surat

Untuk sahabatku Anggi,


8. berikut ini adalah salah satu jenis surat 13. Pendahuluan, isi dan penutup
pribadi, yaitu merupakan 3 unsur yang terbuat dalam
A. kekerabatan 1 nama unsur, yaitu.....
B. tugas A. salam pembuka
C. perizinan B. salam penutup
D. undangan C. alamat surat
E. lamaran kerja D. isi surat
E. tanggal

9. Bahasa yang digunakan dalam surat


pribadi adalah ragam bahasa 14. Surat pribadi dibedakan menjadi
A. santai A. 4
B. kasar B. 3
C. resmi C. 2
D. baku D. 5
E. formal E. 3

10. Unsur unsur surat pribadi ada


A. 6
B. 8
C. 10
D. 4
E. 9

11. yg perlu diperhatikan dalam surat


pribadi yaitu.....
A. penulisannya
B. jenis suratnya
C. etika dan sopan santunnya
D. nama pengirimnya
E. tanda tangan

12. "pengirim" dalam surat pribadi adalah


orang yang......
A. membaca
B. menulis
C. memeriksa
D. mengungkapkan
E. melupakan
PAKET 11 3. Bacalah dua pantun berikut
Pantun 1
soal gurindam, pantun, majas Orang naik menjerat rusa
Rusa digulai diberi sambal
1. Perhatikan pernyataan berikut!
Orang baik ingat usia
1) dalam satu bait terdiri dari
Usia dipakai mencari bekal
empat baris.
Pantun 2
2) semua baris merupakan isi.
Daun terap di atas dulang
3) berima sama.
Anak udang mati di tuba
4) berisi nasihat dan pesan
Dalam kitab ada terlarang
moral.
Yang haram jangan dicoba
5) baris 3-4 adalah isi.
Perbedaan isi pantun tersebut adalah
pernyataan yang benar tentang ….
gurindam adalah... A. Pantun 1: berpesan agar kita
membawa bekal Pantun 2:
A. 1, 2, 3 berpesan agar tidak melakukan hal
B. 2, 3, 4 yang haram
C. 3, 4, 5 B. Pantun 1: menjerat rusa dapat
D. 4, 5, 1 diguna-kan sebagai bekal Pantun
E. 5, 1, 2 2: kita dapat mencari udang di
dulang
2. Berikut ini yakni pola gurindam yang C. Pantun 1: daging rusa enak
sempurna terdapat adalah…. dimakan jika diberi sambal Pantun
A. Buah duku buah durian 2: kitab terlarang tidak boleh
Hanya kau kesayangan dibaca
B. Buah duku buah durian D. Pantun 1: gulai rusa sangat baik
Durian matang lezat sekali untuk orang lanjut usia Pantun 2:
C. Jika ingin jadi anak pintar hal yang haram tidak boleh dicoba-
Setiap ketika harus belajar coba
D. Jika ingin menjadi mulia E. Pantun 1: hidup hendaknya
Ingatlah selalu menjadi baik digunakan untuk mencari bekal ke
E. Siang hujan siang panas akhirat Pantun 2: kita tidak boleh
Aku maman kamu rinas melakukan hal yang terlarang
Penjelasan : awaban a salah.
4. Di bawah ini yang termasuk majas
Meskipun rima kesudahannya sama, namun
penegasan, kecuali…..
alasannya baris pertama bukan isi, berarti bukan
A. Majas pararelisme.
gurindam. Sementara, gurindam yakni dua-
B. Majas klimaks.
duanya yakni isi. Jawaban b terang salah. Rima
C. Majas retorik.
tidak sama. Itu cocoknya sebagai sampiran
D. Majas hiperbola.
sebuah pantun. Jawaban d salah alasannya
E. Majas taulogi.
meskipun sama-sama merupakan isi, tapi rima
kedua baris tersebut tidak sama. Dua baris
5. Bacalah puisi berikut.
tersebut lebih cocok dijadikan sebagai isi
Saat bebatuan mulai memanggilmu.
sebuah pantun.
Tersenyum manis kala bersentuhan C. hindarkan sifat ingin
dengan alam. mengetahui kekurangan orang
Yang tak akan bosan menyapamu. lain
Awan pun akan menangis merindukan D. lebih baik mengoreksi diri
hadirmu. sendiri daripada
membicarakan kejelekan
Majas yang terkandung dalam puisi orang
diatas adalah…… E. kejelekan diri sendiri tidak
A. Majas personifikasi. pantas dibicarakan dengan
B. Majas metonimia orang lain
C. Majas hiperbola.
D. Majas perumpamaan. 8. Bacalah kutipan cerpen berikut
E. Majas pararelisme.
Seseorang yang mencintaimu karena
fisik, maka suatu hari ia juga akan pergi
6. Bacalah puisi dibawah ini!
karena alasan fisik tersebut. Seseorang
Memang salahku. yang menyukaimu karena materi, maka
Buaya darat kepercayai. suatu hari ia juga akan pergi karena
Memang tak bisa dipungkiri melati kini materi. Tetapi seseorang yang
telah ternodai. mencintaimu karena hati, maka ia tidak
Memang salahku. akan pernah pergi! Karena hati tidak
Kepercayai bonglon itu. pernah mengajarkan tentang ukuran
relatif lebih baik atau lebih buruk.
Dan tak kusangka dia berpindah
Tere liye (berjuta rasanya)
warna. Majas yang terkandung dalam kutipan
tersebut adalah…
Jenis majas yang ada dalam puisi A. Majas simbolik
diatas adalah….. B. Majas ironi
A. Majas metafora. C. Majas personifikasi
B. Majas simile. D. Majas sarkasme
C. Majas repetisi. E. Majas litotes
D. Majas antiklimaks.
E. Majas simbolik. 9.Bacalah pantun berikut!
Orang banyak pergi ke bali
7. Bacalah gurindam berikut Ke makassar jalan ke kali
Amal baik sejutakali
Keaiban orang jangan dibuka Balasannya saat mati
Keaiban sendiri hendaklah sangka
Burung gelatik pergi cukur
Maksud gurindam tersebut adalah …. Cukurnya di kota Malang
A. tidak baik membuka Adik cantik berbudi luhur
kesalahan diri sendiri dan Membuat hamba mabuk
usahakan memperbaikinya kepayang
B. tidak baik membicarakan Perbedaan jenis pantun tersebut
kesalahan orang lain di adalah ....
hadapan umum
Pantun 1 Pantun 2
A Dewasa anak muda
12. Perhatikanlah contoh majas berikut
B. Duka Menyenangkan
1) Banyak tikus berkeliaran di gedung
C. Nasihat Jenaka rakyat. (tikus disini merupakan
simbol bagi para koruptor).
D. teka-teki Nasihat 2) Apalah dayaku yang hanya
E. Dewasa Gembira seorang pelayan.
3) Akan kudaki gunung hanya untuk
melihatmu.
10. Bacalah pantun berikut! 4) Engkau ibarat rembulan mala mini.
Tanah liat tanah yang mati 5) Dia merasa sepih di tengah
Dibuat orang bahan mainan keramaian.

Karena itu jangan abaikan Majas Litotes terdapat pada
Larik yang tepat untuk melengkapi nomor…
pantun tersebut adalah .... A. 5)
B. 4)
A. Ditata memanjang agar rapi
C. 3)
B. Hidup sehat perlu
D. 2)
diupayakan
E. 1)
C. Dibentuk dirupa dengan hati
D. Sehat tubuh sangatlah
berarti
E. Badan yang kuat yang 13. Cermati pantun berikut!
diperlukan Di sini kosong di sana
kosong,
11. Bacalah kutipan cerpen berikut! tak ada batang
tembakau.
1) Susah sekali cari pacar. 2) Bukan saya berkata
aku ingin punya kekasih bule, bohong,
dengan rambut pirang, hidung
….
mancung dan mata biru. 3)
wuah itu jenis yang langka di Kalimat yang tepat untuk
Hindia Belanda. 4) kawan-
melengkapi pantun tersebut
kawan ku di benteng,
beberapa ada yang nekat adalah ....
mengambil kekasih gadis- A. Mari kita bersenda gurau.
gadis pribumi. 5) huh! Hitam, B. Saya selalu isap tembakau
dekil, bodoh,..aku sendiri tidak C. Sama-sama menjadi risau
berselerah. D. Semua teman memuji engkau
E. Setiap saat ada yang pantau
Kalimat nomor berapakah yang
merupakan majas sarkasme?
A. 1) 14. Cermati kutipan gurindam berikut!
B. 2) Mengupat dan memuji
C. 3) hendaklah pikir
D. 4) Di situ banyak orang yang
E. 5) tergelincir
Isi gurindam tersebut adalah.... .

A. Berpikirlah sebelum
memuji dan mencela
orang lain agar terhindar
dari kesalahpahaman.
B. Tindakan mencela dan
memuji selalu
mengakibatkan
penyesalan diri sendiri di
kemudian hari.
C. Mencela dan memuji
merupakan dua
perbuatan yang tidak
dapat dipisahkan, dua
sifat yang saling
berhubungan
D. Baik celaan maupun
pujian tidak perlu kita
pikirkan secara
berlebihan karena tidak
bermanfaat.
E. Mencela dan memuji
seseorang merupakan
perbuatan yang dapat
menjerumuskan diri kita.

15. Majas retorik merupakan jenis majas…


A. Majas perbandingan
B. Majas pertentangan
C. Majas sindiran
D. Majas penegasan
E. Majas percintaan

Penjelasan : karena majas retorik


adalah Majas retorik merupakan majas
atau gaya bahasa untuk menanyakan
sesuatu yang jawabannya telah
terkandung dalam pertanyaan tersebut.
Dan merupakan dari jenis majas
penegasan.
PAKET 12 C. J.S.Badudu. 2000. Cinta yang
Terlarang. Bandung: Pustaka Prima
1. Judul buku : Tata Bahasa Baku
Indonesia; D. Badudu, J.S. 2000. Cinta yang
Penulis : Abdul Wahab; Terlarang. Pustaka Prima: Bandung

Penerbit : Erlangga Yogyakarta; E. Badudu, J.S. 2000. Cinta yang


Terlarang. Bandung : Pustaka Prima
Tahun terbit : 2001.

3. Judul buku : Sayuran Hidroponik di


Penulisan daftar pustaka dari data buku Halaman Rumah
tersebut di atas ialah…. Pengarang : Fransisca Wungu Prasasti
A. Abdul Wahab. 2001. Tata Bahasa Penerbit : Gramedia
Baku Indonesia. Yogyakarta: Erlangga.
B. Wahab, Abdul. 2001. Tata Bahasa Tahun terbit : 2008
Baku Indonesia. Yogyakarta: Erlangga.
C. Wahab, Abdul, 2001. Tata Bahasa Kota terbit : Jakarta
Baku Indonesia. Yogyakarta: Erlangga.
D. Wahab, Abdul, Tata Bahasa Baku
Indonesia, (Yogyakarta: Erlangga), Bila Mawar hendak mencantumkan
2001. identitas buku tempat ia mengutip pada
E. Abdul Wahab, Tata Bahasa Baku daftar pustaka, penulisan daftar pustaka
Indonesia (Yogyakarta: Erlangga), yang tepat adalah…..
2001.
A. Fransisca Wungu Prasasti. 2008.
Sayuran Hidroponik di Halaman
Rumah. Jakarta: Gramedia.
B. Fransisca Wungu Prasasti. 2008.
2. Judul buku : Cinta yang Terlarang Sayuran Hidroponik di Halaman
Pengarang : J.S. Badudu Rumah. Jakarta: Gramedia.
Penerbit : Pustaka Prima C. Prasasti, Fransisca Wungu. 2008.
Sayuran Hidroponik di Halaman
Tahun terbit : 2000 Rumah. Jakarta: Gramedia.
D. Prasasti, Fransisca Wungu. 2008.
Kota terbit : Bandung Sayuran Hidroponik di Halaman
Rumah. Jakarta : Gramedia.
E. Prasasti, Fransisca Wungu. 2008.
Penulisan daftar pustaka yang benar “Sayuran Hidroponik di Halaman
adalah…. Rumah.” Jakarta: Gramedia.

A. Badudu, J.S. 2000. Cinta yang


Terlarang. Bandung : Pustaka Prima 4. Perhatikan identitas buku berikut!
Pengarang : Abdul Rozak
B. J.S.Badudu. 2000. Cinta yang
Terlarang. Bandung: Pustaka Prima Judul Buku : Kalimat Efektif
Penerbit : PT Gramedia Ansana, adalah pecinta alam. Maka, tidak
menherankan apabila anaknya, Atik
Kota tempat terbit : Jakarta kemudian menjadi ahli biologi. Atik senang
buku, ia satu dengan buku. Ia membuat
Tahun terbit : 1986
karirnya dengan buku. Keistimewaan
Mangunwijaya lagi bahwa ia menampilkan
penutur-penutur sesuai dengan tingkat
Penulisan daftar pustaka yang tepat sosial dan lingkungannya.
berdasarkan identitas buku tersebut
adalah . . . .
Masalah yang disoroti dalam penggalan
A. Rozak, Abdul. Kalimat Efektif. Jakarta: resensi novel di atas adalah . .
PT Gramedia. 1986.
B. Rozak, Abdul. 1986. Kalimat Efektif. A. Kelebihan pengarang (Mangunwijaya)
Jakarta: PT Gramedia. dalam menggambarkan latar.
C. Abdul, Rozak. 1986. Kalimat Efektif. B. Cara Mangunwijaya bercerita dalam
Jakarta: PT Gramedia. novel.
D. Abdul, Rozak. Kalimat Efektif. Jakarta: C. Latar belakang kehidupan tokoh Atik.
PT Gramedia. 1986. D. Pemaparan keturunan tokoh Atik.
E. Abdul, Rozak, Kalimat Efektif. Jakarta: E. Kepandaian Mangunwijaya dalam
PT Gramedia. 1986. menulis cerita.

5. Lina berkeinginan untuk mencantumkan 7. Terlepas dari berbagai


identitas buku yang ia kutip dalam catatan ketidaksempurnaannya, harus diakui
kaki, penulisan catatan kaki yang benar bahwa buku pertama seorang “yogi buku”
adalah… ini merupakan karya yang memikat.Bahkan
A. Fransisca Wungu Prasasti, “Sayuran cara dan gaya pengungkapannya, dalam
Hidroponik di Halaman Rumah” kadar tertentu, telah memberikan sentuhan
(Jakarta:Gramedia, 2008), hlm.19. sastra yang cukup enak dinikmati. Kita
B. Fransisca Wungu Prasasti, Sayuran menantikan karya berikutnya. Sumber:
Hidroponik di Halaman Rumah Majalah Matabaca, Agustus 2002.
(Jakarta:Gramedia, 2008), hlm.19.
C. Fransisca Wungu Prasasti, Sayuran
Hidroponik di Halaman Rumah Pernyataan yang tepat untuk penutup
(Jakarta:Gramedia, 2008), hlm.19. resensi tersebut adalah . .
D. Prasasti, Wungu Fransiska, Sayuran A. Penutup resensi buku tersebut sangat
Hidroponik di Halaman Rumah tepat.
(Jakarta:Gramedia, 2008), hlm.19. B. Pada bagian penutup resensi buku
E. Prasasti, Wungu Fransiska, Sayuran seharusnya mengajak untuk membaca
Hidroponik di Halaman Rumah buku karena sangat bermanfaat.
(Jakarta:Gramedia, 2008), hlm.19. C. Pada bagian penutup resensi buku
seharusnya mengajak pembaca untuk
memikirkan, merenungkan, dan
6. Betapa apiknya Mangunwijaya menelusuri mendiskusikan lebih jauh fenomena
latar keturunan tokoh Atik. Ayahnya, Pak atau problema yang muncul dalam
sebuah buku. Setelah dibaca, ternyata buku tersebut
D. Pada bagian penutup resensi buku menggunakan bahasa yang mudah
seharusnya memberikan informasi dipahami.
atau pemahaman yang komprehensif
tentang apa yang tampak dan
terungkap dalam sebuah buku. Kalimat resensi yang tepat untuk
E. Pada bagian penutup resensi buku menggambarkannya adalah . .
seharusnya berisi uraian tentang
buku itu penting untuk siapa dan A. Isi buku ini bercerita tentang keadilan
mengapa. dan kebijaksanaan Nabi Daud. Daud
menjadi raja sudah kehendak dari
Tuhan.
8. Cerita ini baik dan mudah ditangkap. B. Bahasa yang digunakan dalam buku ini
Pengarang menyajikan masalah yang aktual mudah dipahami dan komunikatif.
dan sering kita jumpai sehari-hari. Dengan demikian, pembaca dapat
Semuanya dapat diterima akal sehat serta menikmati cerita tanpa harus berpikir
tidak membosankan. Pengarang keras.
menguraikan panjang lebar karakter tokoh- C. Selain menarik, kelebihan buku ini juga
tokohnya. Jadi cerita tidak kabur. Sayang dilengkapi dengan gambar yang
pengarang sering mengulang-ulang kata mendukung isi cerita sehingga mudah
porno. Dalam satu buku, mungkin lebih dari dimengerti.
empat kisah. D. Kekurangan cerita ini hanya terletak
pada kisahnya yang terlalu singkat
Kalimat resensi yang mengungkapkan sehingga ceritanya kurang lengkap
keunggulan buku adalah . . E. Cerita dalam buku ini sangat menarik,
dapat membuat kita seolah-olah hadir
A. Cerita dapat diterima akal sehat dan pada masa nabi terdahulu
tidak membosankan
B. Karakter tidak tergambar dengan cepat 10. Sulit sekali menemukan kekurangan pada
dan membaca berulang-ulang buku ini. Semua unsur yang seharusnya
C. Pengarang menghidupkan cerita dimiliki sebuah karya fiksi terpenuhi dalam
dengan cerita porno dan menghalangi buku ini. Bagi siswa yang tidak senang
D. Cerita diungkapkan seperti lazimnya membaca karya sastra memang buku ini
cerita yang lain tidak begitu menarik sebab novel ini serius
E. Kebosanan dapat diatasi oleh pembaca dan tidak cukup menghibur.
buku ini

Masalah yang dinilai dalam penggalan


9. Judul : Riwayat Nabi Daud (Raja resensi di atas adalah . .
Adil Bijaksana, Nabi yang Mulia)
Pengarang : Ismail Pamungkas A. Kekurangan/kelemahan novel tersebut
B. Keunggulan/kelebihan novel tersebut
Penerbit : PT Remaja Rosdakarya, C. Latar belakang pengarang novel
Bandung D. Kekurangan dan kelebihan novel
tersebut
Tahun : 1995 E. Sinopsis novel tersebut
bukan sebagai hal yang asing. Itulah
11. "Insiden" kecil ini mewarnai karya pendapat Dhany.
Suwarno, yang kalau tidak salah juga
merupakan disertasinya. Suwarno adalah 13. Mawar hendak memasukkan kutipan
orang Yogya. Pendeknya, ia hidup di sana tersebut ke dalam karya tulis yang
dan merasakan demikian membudaya, dibuatnya, penulisan kutipan langsung
Suwarno mengagumi Hamengku Buwono yang paling tepat adalah….
IX. Akibatnya, buku itu over-repetitive, A. Menurut Fransisca (2008:19), tanaman
kurang bermutu, dan kurang perbandingan. yang ditanam dengan cara hidroponik
pada prinsipnya tidak boleh
kekurangan air.
Penggalan resensi di atas merupakan B. Menurut Prasasti (2008:19), tanaman
unsur resensi yang menunjukkan . . yang ditanam dengan cara hidroponik
pada prinsipnya tidak boleh
A. Keunggulan buku
kekurangan air.
B. Isi pokok buku
C. Tanaman yang ditanam dengan cara
C. Deskripsi buku
hidroponik pada prinsipnya tidak boleh
D. Kelemahan buku
kekurangan air (Prasasti, 2008:19)..
E. Identitas buku
D. “Tanaman yang ditanam dengan cara
hidroponik pada prinsipnya tidak boleh
kekurangan air” (Fransisca, 2008:19).
12. “Di dalam buku yang berjudul Demokrasi
E. “Tanaman yang ditanam dengan cara
Pancasila, halaman 45, terbitan tahun
hidroponik pada prinsipnya tidak boleh
1999, oleh penerbit Suara Indonesia,
kekurangan air” (Prasasti, 2008:19).
Bandung, Wakhid Dhany berpendapat,
"Bagi bangsa Indonesia demokrasi
bukanlah sesuatu yang asing." Kutipan langsung identik dengan
Jika pendapat itu dikutip oleh orang lain, penggunaan tanda petik dan diakhiri
pengutipannya dilakukan dengan cara dengan nama penulis, tahun, dan halaman.
seperti
berikut ini, kecuali ...
A. Menurut Dhany (1999:45) demokrasi 14. Hawari menuliskan pendapatnya dalam
bukanlah hal yang asing bagi bangsa bukunya yang berjudul Metode Penelitian
Indonesia. Bidang Sosial yang terbit pada tahun 2011
B. Diungkapkan oleh Dhany (1999:45) halaman 21, bahwa pengetahuan yang
bahwa demokrasi bukanlah hal yang disampaikan oleh Tuhan merupakan
asing bagi bangsa Indonesia. kebenaran yang tidak perlu diragukan lagi.
C. Bangsa Indonesia memandang Pendapat tersebut dikutip oleh Candra
demokrasi bukan sesuatu yang asing Kirana, seorang mahasiswi tingkat akhir
(Dhany, sebuah PTN di Semarang, yang sedang
1999:45). menyelesaikan skripsinya.
D. Pendapat Dhany (1999:45) Berdasarkan informasi di atas, penulisan
menyebutkan bahwa demokrasi bagi kutipan yang benar adalah :
bangsa Indonesia tidak dipandang
sebagai hal yang asing. A. Tidak semua masalah dapat
E. Pada tahun (1999:45) bangsa dipecahkan dengan kemampuan
Indonesia menganggap demokrasi berpikir dan hati nurani manusia. Oleh
karena itu, manusia memerlukan adalah :
sumber kebenaran yang berupa wahyu A. Pertama, komposisi program studi di
Tuhan. “Pengetahuan yang kampus tidak sesuai dengan variasi
disampaikan oleh Tuhan itu merupakan kebutuhan tenaga kerja di lapangan.
kebenaran yang tidak perlu diragukan Kedua, terjadi degradasi mutu
lagi” (Hawari, 2011, 21). lulusan. (Raharjo: 2009:1)
B. Tidak semua masalah dapat B. Menurut Yusuf Efendi (2009: 56)
dipecahkan dengan kemampuan mengemukakan bahwa bekerja tidak
berpikir dan hati nurani manusia. Oleh bisa dilepaskan dari kehidupan
karena itu, manusia memerlukan manusia, baik secara fisik maupun
sumber kebenaran yang berupa wahyu psikologis.
Tuhan. “Pengetahuan yang C. Argumentasi itu tidak lain dari pada
disampaikan oleh Tuhan itu merupakan usaha untuk mengajukan bukti-bukti
kebenaran yang tidak perlu diragukan atau menentukan kemungkinan-
lagi” (Hawari, 2011: 21). kemungkinan untuk menyatakan
C. Tidak semua masalah dapat sikap atau pendapat mengenai suatu
dipecahkan dengan kemampuan hal.
berpikir dan hati nurani manusia. Oleh D. Gorys Keraf (Komposisi: 1989 hlm. 3)
karena itu, manusia memerlukan menjelaskan bahwa argumentasi
sumber kebenaran yang berupa wahyu adalah suatu retorika yang berusaha
Tuhan. (Pengetahuan yang untuk memengaruhi sikap dan
disampaikan oleh Tuhan itu merupakan pendapat orang lain.
kebenaran yang tidak perlu diragukan E. Untuk memperkuat jalinan cerita
lagi Hawari, 2011, 21). dapat menggunakan alur,
D. “Tidak semua masalah dapat penokohan, bahasa, dan simbol-
dipecahkan dengan kemampuan simbol yang dipakai oleh pengarang
berpikir dan hati nurani manusia. Oleh (Imam Syare’iedan A. Syukur
karena itu, manusia memerlukan Ghazali, 1995, hlm 10).
sumber kebenaran yang berupa wahyu
Tuhan. Pengetahuan yang
disampaikan oleh Tuhan itu merupakan
kebenaran yang tidak perlu diragukan
lagi” (Metode Penelitian Bidang Sosial,
Hawari, 2011:21).
E. Tidak semua masalah dapat
dipecahkan dengan kemampuan
berpikir dan hati nurani manusia. Oleh
karena itu, manusia memerlukan
sumber kebenaran yang berupa wahyu
Tuhan. “Pengetahuan yang
disampaikan oleh Tuhan itu merupakan
kebenaran yang tidak perlu diragukan
lagi”. (Metode Penelitian Bidang Sosial,
Hawari: 2011:21).

15. Penulisan kutipan berikut yang benar

Anda mungkin juga menyukai